ROSH missed questions

Lakukan tugas rumah & ujian kamu dengan baik sekarang menggunakan Quizwiz!

Criteria for metabolic syndrome

At least THREE characteristics must be present to diagnose metabolic syndrome (ATP III) criteria: ■ Abdominal obesity: Waist CIRCUMFRENCE (>88cm (35in) Woman, (>102 cm (40 in) MEN) ■ HDL (<40mg/dL) (Men) and (<50 mg/dL) (Women) or drug treatment for low HDL cholesterol ■ TRIGLYCERIDE (>150) or drug treatment for elevated triglycerides ■ B/P (HTN) (>130/85 mmHg) or drug treatment for HTN ■ FASTING GLUCOSE (>100mg/dL) (5.6 mmol/L) or drug treatment for elevated blood glucose

A 67-year-old woman presents complaining of dark urine. Over several months she has had increased fatigue and weight loss. There has not been any fever or night sweats. On physical examination, her abdomen is soft, non-distended with a palpable mass in the right upper quadrant that is non-tender. Her laboratory values are notable for a total bilirubin of 6.4 mg/dl and a direct bilirubin of 5.0 mg/dl. Which of the following imaging studies is the next best step in the workup? A) Abdominal CT sc

Correct answer: A) Abdominal CT scan Explanation: The patient's symptoms are concerning for pancreatic cancer. This patient has developed fatigue, weight loss, and an abdominal mass, a classic presentation of someone with a mass at the head of the pancreas compressing the bile duct. Patients will often also complain of weight loss typically due to loss of appetite. The imaging study of choice in this situation is an abdominal CT scan.

A 35-year-old-man with a long history of occasional bloody diarrhea and abdominal pain presents with acute-onset severe abdominal pain. Vital signs are significant for a temperature of 102.2°F (39°C), heart rate 140 bpm, and blood pressure 82/55 mm Hg. On physical exam, his abdomen is distended and tympanitic. Which of the following diagnostic studies is indicated first at this time? A) Abdominal radiograph B) Barium enema C) CT scan of the abdomen/pelvis with IV contrast D) Sigmoidoscopy

Correct answer: A) Abdominal radiograph Explanation: The patient's presentation is suspicious for toxic megacolon, a complication of inflammatory bowel disease. The hallmark of toxic megacolon is colonic dilatation in a patient with a known inflammatory condition of the colon who appears systemically toxic. Patients usually have experienced symptoms of colitis for several days before the onset of toxic megacolon. Diagnosis is made with an abdominal radiograph that reveals dilatation of the colon with a diameter > 6 cm. The presence of inflammation and toxicity differentiates toxic megacolon from other disorders that cause colonic dilatation (mechanical obstruction, pseudo-obstruction, and congenital megacolon).

Which of the following is the most common cause of acute pancreatitis? A) Biliary tract pathology B) ERCP C) Ethanol ingestion D) Medication side effects

Correct answer: A) Biliary tract pathology Explanation: Biliary tract pathology (gallstones) is the leading cause of acute pancreatitis, accounting for approximately 45% of cases. The incidence may be as high as 66% in some regions. Stones from the bile duct, pancreatic duct, or common bile duct can obstruct the pancreatic duct, resulting in bile reflux, increased pancreatic secretions, and activation of pancreatic enzymes. Patients classically present following ingestion of a fatty meal or after binge drinking and complain of epigastric pain, nausea, and vomiting. The epigastric pain is constant in nature with radiation directly into the back and is often eased when the patient leans forward. Because of the retroperitoneal location of the pancreas, however, rebound is generally absent.

A patient with irritable bowel syndrome complains mainly of lower abdominal pain. She denies diarrhea or constipation, and rarely has problematic flatulence. For this patient's abdominal pain, which of the following medications do you recommend? A) Desipramine B) Loperamide C) Lubiprostone D) Rifaximin

Correct answer: A) Desipramine Explanation: Irritable bowel syndrome is considered an intestinal motility disorder in which chronic diarrhea is the main symptom. Associated symptoms include abdominal pain, bloating, flatulence and constipation. Some dysmotility causes include improper secretion due to luminal or environmental stimuli, decreased pain thresholds (hence, increased pain sensitivity) and dysregulation of the brain-intestinal axis. Rome IV diagnostic criteria are based on symptom presence on at least one symptom, on average, one day per week over the past 3 months plus 2 or more of the following: defecation decreases symptoms, a change in the frequency of stooling or a change in the shape of stool. Treatment regimens are geared toward which of the symptoms a patient with irritable bowel syndrome suffers from most: pain, diarrhea, bloating or constipation. The pain associated with irritable bowel syndrome can be treated with selective-serotonin reuptake inhibitors like cita

You diagnose a 43-year-old man with alcohol withdrawal. Lab results reveal a hemoglobin of 12 g/dL and an MCV of 115 fL. Which of the following is the most likely cause of these findings? A) Direct ethanol toxicity B) Pyridoxine deficiency C) Thiamine deficiency D) Vitamin B12 deficiency

Correct answer: A) Direct ethanol toxicity Explanation: Ethanol affects practically every organ system in the body. This patient is suffering from a macrocytic anemia, most likely as a result of chronic alcohol use disorder. Chronic ethanol intake directly suppresses bone marrow by impairing protein synthesis, causing anemia or even pancytopenia. Although patients who have heavy alcohol use often have concomitant nutritional deficiencies that contribute to the development of megaloblastic anemia, the most likely cause of macrocytic anemia is due to direct ethanol toxicity. Macrocytosis is present in the majority of patients with heavy alcohol use even before significant anemia appears. This is typically reversed after several months of abstinence from alcohol. In addition to the harmful effects of ethanol itself, its metabolite, acetaldehyde, is inherently toxic to biologic systems. Patients presenting with acute ethanol intoxication also commonly have decreased serum ionized magnesi

A 25-year-old truck driver presents with a 1-day history of severe rectal pain. Your examination shows a large thrombosed external hemorrhoid. Which one of the following is the preferred initial treatment for this patient? A) Elliptical excision of the thrombosed hemorrhoid B) Infrared coagulation C) Rubber band ligation of the hemorrhoid D) Stool softeners and a topical analgesic

Correct answer: A) Elliptical excision of the thrombosed hemorrhoid Explanation: Hemorrhoids are swollen blood vessels in the lower rectum. External hemorrhoids occur distal to the dentate line and develop as a result of distention and swelling of the external hemorrhoidal venous system. Engorgement of a hemorrhoidal vessel with acute swelling may allow blood to pool and, subsequently, clot; this leads to the acutely thrombosed external hemorrhoid. A patient with a thrombosed external hemorrhoid usually presents with complaints of an acutely painful mass at the rectum. Pain caused by hemorrhoids usually arises only with acute thrombus formation. This pain peaks at 48-72 hours and begins to decline by the 4th day as the thrombus organizes. The appropriate management of a thrombosed hemorrhoid presenting within 72 hours of the onset of symptoms is elliptical excision. Infiltration of a local anesthetic containing epinephrine is followed by elliptical incision and excision of the thromb

A 15-year-old girl presents to the emergency department complaining of headaches. The patient denies fever, sore throat, vomiting, and head trauma. Physical exam reveals facial acne, papilledema, and splenomegaly. Excess of which of the following vitamins is most likely causing her symptoms? A) Vitamin A B) Vitamin B3 C) Vitamin B6 D) Vitamin C E) Vitamin E

Correct answer: A) Vitamin A Explanation: Vitamin A excess can cause acute papilledema, characterized by blurred disc edges, an absent physiologic cup, and intraretinal exudates. It can also cause anorexia, slow growth, drying and cracking skin, enlargement of liver and spleen, bone fragility, swelling and pain of long bones, increased intracranial pressure, alopecia, and carotenemia. Vitamin A (retinol) is a fat-soluble vitamin and component of retinal pigments for vision in dim light. It is also involved in bone and tooth development and formation and maturation of epithelia. Vitamin A can be found in liver, fish, liver oils, whole milk, carotenoids, green vegetables, yellow fruits and vegetables. Vitamin A can be converted to retinoic acid (tretinoin) and isotretinoin, which are used in the treatment of acne and keratosis pilaris. - Hypervitaminosis A (Increased bone turnover) ( Alters metabolism of other fat-soluble vitamins (D,E,K)

Which one of the following groups of lab results is most consistent with a complication that commonly occurs 6-12 weeks after acute hepatitis? A) WBC decreased, RBC decreased, platelets decreased B) WBC decreased, RBC increased, platelets increased C) WBC increased, RBC decreased, platelets decreased D) WBC increased, RBC increased, platelets increased

Correct answer: A) WBC decreased, RBC decreased, Platelet decreased Explanation: Aplastic anemia is a known complication of acute hepatitis. This affects up to 2% of all patients after their initial illness. Hepatitis-associated aplastic anemia most often affects adolescent boys and young men, is most commonly seen 6-12 weeks after hepatitis, and can be fatal if untreated. Its etiology is unclear. There is no known association with blood transfusions, drugs, or toxins. Most patients have been seronegative for hepatitis A, B, and C, but it is believed to be the result of autoimmune bone marrow failure. Lab findings show a decrease in the WBC, RBC, and platelet counts.

An 83-year-old woman with a history of constipation presents to the ED complaining of a rectal mass. On exam, you note the mass seen in the image above. What is the most appropriate next step in management? A) Attempt manual reduction B) Consultation to a colorectal surgeon C) Immediately begin broad-spectrum ABX D) Inject local anesthesia and perform an excisional thrombectomy

Correct answer: Attempt manual reduction Explanation: Rectal prolapse is a disease of the very young and the very old. In adults, it is most commonly seen in older women and is most often associated with excessive straining while defecating, coughing, or sneezing. This is due to laxity of attachment structures and is often accompanied by prolapse of the bladder (cystocele) and uterus. Often, bloody mucous discharge will be seen along with fecal incontinence and dull pain. Manual reduction is usually sufficient for most cases and should be attempted in the ED.

A 55-year-old-man with known alcoholic cirrhosis presents with a fever and abdominal pain. He is mildly confused and has asterixis on exam. His abdomen is distended and tender to palpation. Paracentesis is completed. The ascitic fluid has a neutrophil count of 280/mm3. Which of the following is the most appropriate intravenous antibiotic for treatment of this condition? A) Ampicillin B) Cefotaxime C) Bactrim D) Vancomycin

Correct answer: B) Cefotaxime Explanation: Spontaneous bacterial peritonitis is a life threatening infection occurring in cirrhotic patients. It is a common condition, but with nonspecific signs is difficult to diagnose. The high mortality rate makes it a critical diagnosis to consider in each patient with ascites. When spontaneous bacterial peritonitis is suspected, diagnostic paracentesis is recommended. Once the diagnosis is confirmed by an ascites white blood cell count of > 1000/mm3, a neutrophil count of > 250/mm3, or a positive gram stain, therapy should start with a third generation cephalosporin (e.g. cefotaxime). An alternative empiric treatment option is a fluoroquinolone. The most common bacterial isolates in spontaneous bacterial peritonitis are Escherichia coli, Klebsiella pneumoniae, and Streptococcus spp.

Surgical treatment of peptic ulcer disease is reserved for those patients with complications associated with ulcer lesions, those refractory to medical intervention, or patients with giant (greater than 3 cm) gastric ulcers. Which of the following is considered the most common postsurgical complication experienced by patients undergoing surgical treatment for peptic ulcer disease? A) Blind loop syndrome B) Gastroparesis C) Steatorrhea D) Weight loss

Correct answer: D) Weight loss Explanation: Weight loss is experienced by up to 30% of patients postsurgically. Patients may limit food intake due to early satiety. Distention or discomfort may occur shortly after even a moderate-sized meal as the residual gastric pouch is smaller. Patients should be counseled to eat smaller and more frequent meals. Additionally, weight loss can be a result of other postsurgical complications such as maldigestion or dumping syndrome.

A 74-year-old man with hyperlipidemia, diabetes mellitus, hypertension, and a 25 pack-year history of tobacco use presents to the clinic with a recent onset of severe epigastric abdominal pain that radiates to his back. His current medications include metformin, exenatide, lisinopril, propranolol, simvastatin, aspirin, and niacin. Vital signs include HR 97 bpm, RR 16 breaths/min, BP 138/87 mm Hg, T 98.8°F, SpO2 98% on room air, and an unintentional weight loss of 12 lbs in 1 month. Physical exa

Correct answer: B) Dilation of the pancreatic and CBD Explanation: Pancreatic cancer is the third leading cause of death from cancer in the United States and has the worst survival rate of any cancer. This malignancy is more common in men and older patients, with diagnosis most common in the 65-74 age group. Cigarette smoking is the greatest risk factor for developing pancreatic cancer, with other potential risk factors including high intake of fat or meat and consumption of sugary, fizzy drinks. Hereditary factors and genetic factors that cause familial cancer syndromes, such as BRCA2 or PALB2 germ-line mutations, should prompt further evaluation for early detection of pancreatic cancer. Additionally, associations exist between chronic pancreatitis, diabetes mellitus, and obesity in the development of pancreatic cancer. Neoplasia of the pancreas can be endocrine or exocrine in nature, with the most common and deadly exocrine tumor being pancreatic infiltrating ductal adenocarcinom

A 20-month-old girl presents with abdominal pain and bilious vomiting for the past 2 days. The father reports that she has episodes where she is inconsolable and is drawing her legs up to her chest. Between the episodes, she behaves normally. He noticed blood in the last diaper he changed. Where is the most likely anatomic location of the suspected diagnosis? A) Junction of descending colon and cecum B) Junction of ileum and ascending colon C) Junction of transverse colon and ascending colon D

Correct answer: B) Junction of the ileum and ascending colon Explanation: The presence of bilious vomiting should always raise concern for intestinal obstruction such as intussusception. Intussusception may be caused by a pathological lead point. This is a lesion or variation in the intestine that is trapped by peristalsis and dragged into a distal segment of intestine. This causes bowel obstruction and ischemia. The most common location is ileocolic intussusception and occurs when the terminal ileum telescopes into the ascending colon. It is usually diagnosed via ultrasound, which shows a coiled spring or target lesion representing layers of intestines within the intestine. Surgery is indicated in patients who are acutely ill or show evidence of perforation. Otherwise, patients are treated with non-operative reduction using hydrostatic or pneumatic pressure by enema.

A 56-year-old man presents to the emergency department with right upper quadrant pain that radiates to the right scapula. His pain increases with deep respiration, and he reports associated nausea and multiple episodes of vomiting. His medical history includes hypertension, atherosclerosis, and gout with medical treatment involving lisinopril, atorvastatin, aspirin, and allopurinol. Vital signs include a HR of 104 bpm, RR of 18/min, BP of 156/99 mm Hg, T of 101.9°F, and SpO2 of 98% on room air.

Correct answer: B) Nonvisualization of the gallbladder on delayed images Explanation: Acute cholecystitis is an inflammatory condition involving the gallbladder. This inflammatory response is most often subsequent to an obstruction of the cystic duct by a gallstone, which leads to inflammation secondary to mechanical, chemical, or bacterial factors. The most commonly indicated bacterial organisms involved in acute cholecystitis include Escherichia coli, Klebsiella sp, Streptococcus sp, and Clostridium sp. Clinical signs and symptoms of acute cholecystitis include biliary pain in the right upper quadrant of the abdomen that progressively worsens. This pain may radiate to the interscapular area, right scapula, or shoulder and is exacerbated by peritoneal irritation to include jarring and deep respiration. Other signs and symptoms include fever, anorexia, nausea, vomiting, and, occasionally, jaundice. An enlarged, tense gallbladder may be palpable during physical exam with localized reb

A 38-year-old woman presents with rectal pain and blood on the toilet paper when she wipes after bowel movements for the last two days. Physical examination reveals a mildly tender, external, non-thrombosed hemorrhoid without active bleeding. Which of the following is the most appropriate pharmacotherapy? A) PO Docusate B) Topical hydrocortisone C) Topical lidocaine D) Topical nifedipine

Correct answer: B) Topical hydrocortisone Explanation: A short course of a topical steroid cream (e.g. hydrocortisone) or suppositories used twice daily has been shown to improve pain and diminish swelling associated with external hemorrhoids. Hemorrhoidal cushions are comprised of sub-epithelial connective tissue, elastic tissue, blood vessels, and smooth muscle, and help maintain fecal continence. Abnormal swelling of the cushions can occur secondary to increased pressure that causes dilation and engorgement of the vasculature. Risk factors for increased pressure include constipation, prolonged straining at stool, pregnancy, or prolonged sitting or heavy lifting. The diagnosis is mostly clinical and made with visual inspection. Thrombosed hemorrhoids will be firm and tender to palpation. Thrombosed hemorrhoids may be excised for faster symptom resolution and lower recurrence rates. Surgery may be needed for definitive treatment.

A 25-year-old woman presents to the gastrointestinal clinic for follow-up after being diagnosed with Lynch syndrome. She has a germline mutation in the MSH2 DNA mismatch repair gene, and her mother was diagnosed with colon cancer at 43 years of age. The patient reports she has not had any rectal bleeding or changes in bowel movements. Vital signs today include a HR of 80 bpm, BP of 126/70 mm Hg, RR of 20/min, oxygen saturation of 98% on room air, and T of 98.6°F. Physical examination reveals a

Correct answer: C) Annual colonoscopy starting immediately Explanation: Lynch syndrome is the most common cause of inherited colorectal cancer. It significantly increases the risk of colorectal cancer and endometrial cancer but is also associated with an increased risk of ovarian, stomach, small bowel, and urothelial genitourinary cancers. Lynch syndrome has an autosomal dominant pattern of inheritance and is caused by a germline mutation in one of several different DNA mismatch repair genes, such as MLH1, MSH2, MSH6, and PMS2. Colorectal cancer tumors caused by Lynch syndrome have some unique features compared to sporadic colon cancer tumors. Lynch syndrome causes predominantly right-sided tumors; the adenomas tend to be larger, flatter, and more proximal; progression to carcinoma occurs more rapidly; and there is a higher risk of synchronous or metachronous colorectal cancers. Germline genetic evaluation for Lynch syndrome is recommended in the following situations: colorectal canc

A 53-year-old man presents with weight loss, anorexia, abdominal pain, jaundice and early satiety. Which of the following is the most appropriate initial diagnostic test to diagnose and stage pancreatic cancer? A) Biopsy B) CA 19-9 antigen C) CT scan D) MRI

Correct answer: C) CT scan Explanation: For patients presenting with common symptoms of pancreatic cancer, pancreas protocol CT is the standard for diagnosis and staging. Pancreas protocol CT involves triphasic (arterial, late and venous phases) cross-sectional imaging that allows for enhancement between the parenchyma and adenocarcinoma. Moderate to high risk factors for pancreatic cancer include: family history of pancreatic cancer, hereditary pancreatitis, familial atypical multiple mole melanoma, Peutz-Jueger syndrome, cystic fibrosis and the BRCA-2 gene carrier. Routine screening is not recommended for asymptomatic adults who are at average risk. However, it may be reasonable to consider screening persons at high risk of developing pancreatic ductal adenocarcinoma, such as persons from families with known genetic defects predisposing them to pancreatic cancer. If a pancreatic mass on a CT is identified, subsequent ultrasonography and fine-needle aspiration are indicated for fina

A 21-year-old man presents with a complaint of a painful "lump" near his anus. He reports occasional blood on the toilet paper when he wipes after straining to have a bowel movement. He also complains of intermittent peri-anal itching. On inspection, you note a pink, circular lesion visualized on the anus that is not pedunculated. It is firm and slightly tender. What is the most likely diagnosis? A) Anal abscess B) Anal skin tag C) External hemorrhoid D) Internal hemorrhoid

Correct answer: C) External hemorrhoid Explanation: External hemorrhoids represent dilated hemorrhoidal veins that originate below the dentate line. Combined, internal and external hemorrhoids are the most common cause of lower GI bleeds. External hemorrhoids can be visualized at the anal margin and are typically pink and painless until they become thrombosed. Thrombosis is usually preceded by constipation and straining. A thrombosed external hemorrhoid will appear to be blue or purple in color. Bleeding associated with external hemorrhoids is classically described as bright red blood on the toilet tissue observed after wiping. Treatment options for external hemorrhoids range from increased fiber intake and cortisone suppositories to surgical intervention depending on the severity of the clinical presentation.

A 72-year-old man presents to his primary care PA with his daughter, who is worried about his burps. She worries this is a sign of esophageal cancer, which the patient's brother recently died of. The patient has lost 8 pounds since his last visit 7 months ago. Approximately 2 months ago, he had a bad "chest cold" that took a while to clear. Vital signs are BP of 135/90 mm Hg, HR of 80 bpm, RR of 15/min, oxygen saturation of 98% on room air, and T of 98.1°F. The patient says he "just doe

Correct answer: C) Irregularity above the cricopharyngeal muscle Explanation: Zenker diverticulum is a rare herniation of the mucosal and submucosal layers of the esophagus in the Killian triangle, a weak point defined on the sides by the bilateral inferior pharyngeal constrictor muscles and at the base by the cricopharyngeal muscle, which comprises a vital part of the upper esophageal sphincter. Zenker diverticulum appears in older adults who experience signs and symptoms of dysphagia, halitosis, regurgitation of undigested food, and malnourishment, as food, mucus, and even pills become trapped in the pouch. It carries risks for aspiration pneumonia, ulceration and bleeding, and esophageal cancer. Diagnosis is done by esophagography (barium swallow), which is also used to distinguish between various esophageal motility disorders, presenting real-time video of the patient's swallowing. It can detect spasms and stasis as well as ulcerations, narrowings, outpouchings, and strictures.

A young woman suffers from chronic diarrhea. A detailed history provides no evidence of provocative medications as the cause. You send a stool sample to the laboratory and obtain the following results: Color: Yellow-brown Water: High Fat: Negative Blood: Negative WBCs: Negative Osmotic gap: Normal These results suggest which of the following as the most likely causative of this patient's diarrhea? A) Infectious B) Malabsorption C) Motility D) Osmotic

Correct answer: C) Motility Explanation: Chronic diarrhea is diagnosed when a person has 3 or more watery or loose stools per day for greater than 1 month. The difficulty in determining an etiology is due to the extensive differential that accompanies this diagnosis. There are several etiologic classes of chronic diarrhea: medications like SSRIs and PPIs, motility disorders like scleroderma and diabetic autonomic neuropathy, secretory disorders such as that which occurs in laxative abuse, osmotic abnormalities such as lactose intolerance, infectious etiologies like parasitic infections, inflammatory bowel disease and malabsorption abnormalities such as celiac disease and pancreatic insufficiency. Evaluation starts with ruling-out medication causes. Then, a stool sample should be sent for fecal fat, and if present, consider the malabsorptive disorders as causative. If the stool sample contains occult blood or WBCs, consider infectious and inflammatory conditions, then order a stool cu

A 35-year-old meat cutter comes to your office with persistent symptoms of nausea, vomiting, and diarrhea, which began about 36 hours ago on the last day of a 5-day Caribbean cruise. His wife was sick during the first 2 days of the cruise with similar symptoms. Physical examination is negative, and a stool specimen is negative for white blood cells. Which one of the following is the most likely cause of his illness? A) Girardia B) Hepatitis A C) Norovirus D) Rotavirus

Correct answer: C) Norovirus Explanation: Recent reports of epidemics of gastroenteritis on cruise ships are consistent with Norovirus infections due to waterborne or foodborne spread. In the United States these viruses are responsible for about 90% of all epidemics of nonbacterial gastroenteritis. The noroviruses are common causes of waterborne epidemics of gastroenteritis and have been shown to be responsible for outbreaks in nursing homes, on cruise ships, at summer camps, and in schools. Symptomatic norovirus gastroenteritis typically develops 24-48 hours after ingestion of contaminated food or water or after contact with an infected individual. The onset can be abrupt or gradual, but each episode is short-lived, lasting only 24-72 hours. The absence of fecal leucocytes and occult blood in stool is helpful in ruling out other enteroinvasive infectious diarrhea processes. Stool culture should be performed to exclude infection with bacterial organisms such as Yersinia, Shigella

A woman with chronic constipation and poorly controlled diabetes mellitus presents with acute anal pain that is exacerbated by defecation. Inspection of the anal border reveals no abnormalities. Which of the following is the most likely diagnosis? A) Anal fistula B) Perianal hematoma C) Perirectal abscess D) Thrombosed external hemorrhoid

Correct answer: C) Perirectal abscess Explanation: Any abscess, arising from an infection of the anal sinuses, is called an anorectal abscess, or perianal or perirectal abscess. Sporadic onset is common. A perianal abscess is defined as a simple anorectal abscess There are four main types of perirectal abscesses: perianal, ischiorectal, intersphincteric, and supralevator. Dull or throbbing anal pain, usually acute in onset, is the main symptom. It usually worsens prior to, and lessens after, a bowel movement. Direct visual inspection may miss a deep abscess, therefore, it is important to palpate for such a possibility. Imaging is typically not necessary but CT can be obtained if there is concern for a deep abscess. Treatment mainstays are antibiotics and surgical drainage.

A 23-year-old man with a history of ulcerative colitis presents with abdominal pain and vomiting. On exam, he is febrile with a heart rate of 125 beats per minutes and blood pressure of 92/63 mm Hg. He has diffuse abdominal tenderness and distention. Which of the following imaging studies is most appropriate to diagnose toxic megacolon? A) Colonoscopy B) CT C) Plain radiography D) Ultrasound

Correct answer: C) Plain radiography Explanation: This patient presents with signs and symptoms concerning for toxic megacolon, a complication of infectious colitis or inflammatory bowel disease which is potentially lethal. Patients with toxic megacolon present with signs of systemic toxicity and a nonobstructive colonic dilatation ≥ 6 cm. Diagnostic criteria include evidence on plain radiography of colonic dilatation, any three of the following clinical signs: fever, leukocytosis, anemia, or tachycardia, and any one of the following: dehydration, hypotension, altered mental status, or electrolyte abnormality. The goal of treatment is to restore normal colonic motility and decrease the chance of perforation by reducing the severity of colitis. Initial treatment is medical with complete bowel rest and nasogastric tube placement. Patients should be monitored in the intensive care unit. For patients requiring surgery due to urgent or emergent presentation, subtotal colectomy with end-

A 60-year-old missionary woman is being evaluated for dementia. She also has diarrhea. She was recently treated for tuberculosis with isoniazid. Physical examination shows a symmetric hyperpigmented rash on both arms. Deficiency of which of the following vitamins most likely resulted in this patient's condition? A) Vitamin B1 B) Vitamin B12 C) Vitamin B3 D) Vitamin B9

Correct answer: C) Vitamin B3 Explanation: A patient presenting with dementia, dermatitis, and diarrhea, who was recently treated with isoniazid, strongly suggests niacin (vitamin B3) deficiency. The most characteristic finding of niacin deficiency (pellagra) is the presence of a symmetric hyperpigmented rash. Other findings include diarrhea and neurological symptoms (eg, dementia, insomnia, anxiety, delusions, disorientation). Isoniazid (INH) biochemically competes with niacin and inhibits the conversion of tryptophan to niacin, resulting in niacin deficiency.

An overweight 29-year-old roofer presents with acute groin swelling after lifting an 80-pound sack of shingles. Examination reveals a minimally tender mass in the right scrotum. There is no mass in the proximal thigh or abdomen. Which of the following types of hernia do you most likely suspect? A) Direct inguinal B) Epigastric C) Femoral D) Indirect inguinal

Correct answer: D) Indirect inguinal Explanation: The inguinal canal is formed by the inguinal ligament and the lower abdominal muscles. It is a tube from the abdominal cavity into the scrotum, allowing the testicles a passageway to descend through. It has two openings called the deep (internal) and superficial (external) inguinal rings. Hernia is the general term used to describe the passage of tissue or organ from its cavity of origin into a different body cavity. Hernias are common in the inguinal region. A direct inguinal hernia is passage of the abdominal contents straight through a weakened portion of the abdominal wall, usually occurring medial to the inferior epigastric vessels and superior to the inguinal ligament. An indirect inguinal hernia is the passage of abdominal contents through the internal inguinal ring, through the inguinal canal, and out through the external inguinal ring into the scrotum. Indirect inguinal hernias are more common than the direct type. Some risk

A 36-month-old boy is being evaluated at your office. Parents are concerned because he is short and has evidence of developmental delay with hypotonia. He was born at term with a birth weight of 3250 grams. He has a history of poor suck that required therapy. Recent fluorescence in situ hybridization was positive for a deletion on chromosome 15. This patient is at a higher risk of suffering from which of the following conditions? A) Alzheimer disease B) Congenital heart disease C) Hypocalcemic

Correct answer: D) Obesity Explanation: Prader-Willi syndrome is the most common syndromic form of obesity. The syndrome is caused by absence of expression of the paternally active genes on the long arm of chromosome 15, also known as maternal uniparental disomy. Indications for genetic testing in children ages 2 to 6 years old include hypotonia with a history of poor suck and global developmental delay. Neonatal hypotonia is one of the hallmark features of this syndrome and a poor suck can lead to failure to thrive. Learning difficulties are common and can manifest in temper tantrums, stubbornness, and obsessive-compulsive behaviors that can impede school performance. Some of these behaviors are similar to those found in pervasive developmental disorder (autism spectrum disorder). Food-seeking behaviors may include eating garbage, frozen foods, and stealing resources to obtain food. Decreased ability to vomit and increased tolerance of pain can promote binging on spoiled foods and d

A 65-year-old man with obesity presents to the clinic with intermittent, painless hematochezia for the past week. He has no history of inflammatory bowel disease or hemorrhoids. He smokes cigarettes and reports an overall poor diet with frequent consumption of red meat. Blood pressure is 124/80 mm Hg, pulse is 80 beats per minute, and temperature is 98.8°F. No hemorrhoids are noted on exam and there is no evidence of active bleeding. Initial blood analysis reveals a hemoglobin of 13.6 g/dL and

Correct answer: D) Refer for colonoscopy Explanation: Diverticular bleeding is the most common cause of significant lower gastrointestinal bleeding. The incidence of diverticulosis and diverticulitis increases with age. Risk factors for diverticular disease include high dietary consumption of red meat, low dietary fiber, sedentary lifestyle, BMI > 25 kg/m2, and cigarette smoking. Diverticular bleeding usually presents as painless hematochezia, while diverticulitis usually presents as left lower quadrant abdominal pain and tenderness with leukocytosis. It is uncommon to present with symptoms of both diverticular bleeding and diverticulitis. Colonoscopy is the test of choice when a patient presents with painless hematochezia, to identify the source of the bleeding. The majority of cases of bleeding from diverticulosis resolve spontaneously, however, patients are at risk for recurrent bleeding.

A 57-year-old woman with cirrhosis complains of worsening distension of the abdomen and edema up to her lower legs despite compliance with eating less than 2 grams daily of sodium. She denies abdominal pain, constipation, dyspnea or fevers. On exam, her abdomen is distended and has a notable fluid wave. She has 2+ pitting edema to the level of her ankles. Which medication is the most appropriate next step in the management of this patient? A) Hydrochlorothiazide B) Lactulose C) Omeprazole D) S

Correct answer: D) Spironolactone Explanation: This patient has worsened ascites secondary to portal hypertension as a result of cirrhosis. Portal hypertension leads to ascites through a series of hemodynamic changes resulting in the movement of fluid into the peritoneum. The renin-angiotension-aldosterone system is activated leading to sodium retention and additional fluid accumulation. Spironolactone is an aldosterone antagonist, helping to reverse sodium retention, conserve potassium and achieve diuresis. The starting dose is 100 mg daily, with titration up to 400 mg as needed and as tolerated. Side effects of spironolactone are many and may include decreased libido, impotence and gynecomastia. Some patients with ascites secondary to cirrhosis may be managed without diuretic therapy by initiation of a diet including less than 2 grams of sodium per day. Patients with ascites refractory to sodium restricted diets are often placed on combination therapy with both spironolactone and f

Which of the following disorders is associated with celiac disease? A) Medullary thyroid cancer B) Parathyroid hyperplasia C) Pheochromocytoma D) Type 1 DM

Correct answer: D) Type 1 DM Explanation: Type 1 diabetes mellitus is a disorder associated with celiac disease. Conditions which have autoimmune features such as type 1 diabetes mellitus, thyroid disease, juvenile rheumatoid arthritis, dermatitis herpetiformis and autoimmune liver disease are commonly associated with celiac disease. Additionally, celiac disease is also associated with Down syndrome, small bowel lymphoma, and selective IgA deficiency. Celiac disease occurs secondary to ingestion of wheat gluten or related rye and barley proteins in genetically predisposed persons. Characteristic findings include chronic diarrhea or steatorrhea, bloating, weight loss and abdominal pain, pruritic papulovesicular rash on the extensor surfaces called dermatitis herpetiformis, iron and fat soluble vitamin deficiencies and osteoporosis.

A 10-month-old boy is being evaluated for bilious vomiting, diarrhea, and rectal bleeding. There is no fever or history of exposure to allergens or toxins. He appears to be in severe distress and is continuously crying. He is also pulling his legs up to his abdomen, and a small mass can be palpated in the right middle abdomen. Which of the following is the most appropriate next step in management? A) Abdominal CT scan B) Abdominal ultrasound C) Abdominal X-ray D) Contrast enema

Correct answer: D) contrast enema Explanation: Based on the constellation of findings, this patient is at high clinical likelihood of intussusception. Intussusception occurs when one portion of the intestine invaginates into another portion. The most common form is intussusception of the terminal ileum into the right colon (ileocolic intussusception). Intussusception is the most common abdominal emergency in early childhood. Patients with a typical presentation include an infant or toddler with sudden onset of intermittent severe abdominal pain with or without rectal bleeding (currant jelly stools), in the setting of a normal temperature and lack of exposure to toxins or allergens. The patient may be seen pulling the legs up to the abdomen, and a sausage-shaped mass may be palpated in the right upper or middle abdomen. These patients with a high suspicion for intusssusception can proceed directly to nonoperative reduction using a contrast enema (using either barium or air). In these

A 59-year-old man presents to the ED with 12 hours of emesis and abdominal pain. Vital signs include a T of 38.2°C, BP of 110/79 mm Hg, and HR of 109 bpm. On exam, you note a tender 2 x 2 cm bulge with erythema in the abdominal midline above the umbilicus. There is abdominal distension, and an occasional high-pitched bowel sound is heard. After placing an IV line and nasogastric tube and administering broad-spectrum antibiotics, which of the following is the most appropriate course of managemen

Correct answer: D) obtain emergent surgical consultation Explanation: This clinical scenario is consistent with a strangulated loop of bowel (emesis, abdominal pain, fever, erythema, abdominal distension, high-pitched bowel sounds) incarcerated within a ventral hernia. If the contents of a hernia can be returned to their natural cavity by manual reduction, the hernia is termed reducible. If they cannot, it is termed irreducible or incarcerated. Incarcerated hernias are subject to inflammatory and edematous changes and are at risk for strangulation, which refers to vascular compromise of the incarcerated contents. When strangulation is not emergently relieved, necrosis and gangrene develop. The treatment for an incarcerated hernia is surgical fixation. If strangulation is suspected or shock is present, broad-spectrum antibiotics and fluid resuscitation are necessary, but manual reduction is contraindicated because the patient requires emergent surgical care. The reintroduction of isch

A 60-year-old man undergoes a routine colonoscopy. He is asymptomatic and reports no bloody stools, abdominal pain, or recent weight loss. He has struggled with hard stools for years and takes a daily fiber supplement with water. His medical history includes hypertension, and he takes a lisinopril 10 mg every morning. Vitals are BP 126/76 mm Hg, temperature 98.7°F, pulse 89 bpm, and RR 13 breaths/minute, and his physical exam is normal. His colonoscopy shows a 0.9 mm polyp in the rectosigmoid r

Correct answer: E) Villous adenoma Explanation: Routine colonoscopies are used to screen for colon cancer. Colon cancer is often asymptomatic, so regular screening is recommended. Though polyps are commonly seen, many of them benign, a villous adenoma found in the colon requires close surveillance and follow-up. Most colon cancers arise from adenomas of the colon. Though most small adenomas (< 1 cm) are at a low risk for malignancy, polyps that are large or contain villous characteristics are at a higher risk of malignancy and need close surveillance.

A 53-year-old man presents to the clinic for evaluation of a painless "bulge" in his groin. He noticed the protrusion while attempting to sit up from a supine position. His medical history includes hypertension and atherosclerosis, for which he takes enalapril, simvastatin, and aspirin. Vital signs include HR 76 bpm, RR 16 breaths/min, BP 121/77 mm Hg, T 98.7°F, and SpO2 99% on room air. Physical examination reveals a patient who is endomorphic and in no apparent distress. There are no surg

Correct answer: E) Weakness of the transversalis fascia Explanation: Hernias are defined as abnormal protrusions, bulges, or projections of organs or organ portions through defects in the anatomic body cavity in which they are normally contained. Inguinal and femoral hernias appear in the groin, while abdominal wall hernias include umbilical, epigastric, and ventral hernias, among others. A hernia consists of covering tissues (skin, subcutaneous tissue), a peritoneal sac, and various abdominal contents, such as preperitoneal fat, omental fat, and visceral organs. Bowel that becomes entrapped within a hernia can become strangulated or obstructed, leading to further tissue damage and necrosis. Hernias are classified as reducible when the contents of the sac can be returned to the abdominal cavity with manual pressure, unlike the contents of incarcerated or irreducible hernias, which cannot be returned to the abdomen, usually secondary to a narrow hernia neck. An indirect inguinal hernia

A 60-year-old woman with a history of well-controlled diabetes mellitus presents to your office with a complaint of constipation. She has a bowel movement about twice per week with straining and hard stools. She has had intermittent constipation for many years, but over the past four months it has been constant. Which of the following is the most appropriate initial management? A) Fecal disimpaction B) Fiber restricted diet C) Increased fiber intake D) Suppository use as needed

Correct answer: Increased fiber intake Explanation: Constipation is the most common gastrointestinal complaint in the United States and can be divided into two groups: primary or secondary constipation. Primary constipation is due to either functional or idiopathic etiology and consists of normal-transit constipation, slow-transit constipation, and pelvic floor dysfunction. Secondary constipation is caused by systemic diseases such as diabetes mellitus, dietary issues including inadequate fluid intake or structural causes, such as hemorrhoids and anal fissures. Diagnosis is with the Rome IV criteria which involves the patient experiencing at least two of the following symptoms for the past three months or more: straining, < 3 bowel movements per week, manual maneuvering in order to defecate, hard stools, and sensation of either anorectal obstruction or incomplete defecation and loose stools only with laxative use, does not meet the criteria for irritable bowel syndrome. Initial manag

In addition to diverticulitis, what are some other contraindications for a barium enema?

Severe ulcerative colitis, pregnancy, toxic megacolon, and acute abdominal pain.

Vitamin deficiency symptoms?

Vitamin Deficiencies A: night vision loss, xerophthalmia, dry skin (xerosis), growth retardation, Bitot spots on the conjunctivae B1 (thiamine): beriberi, Wernicke-Korsakoff syndrome, cardiac failure; alcoholism, malnutrition B2 (riboflavin): cheilosis, corneal vascularization (the two Cs of B2) B3 (niacin): dermatitis, dementia, diarrhea; corn-based diet (pellagra) B6 (pyridoxine): sideroblastic anemia, convulsions, peripheral neuropathy; INH use B12 (cobalamin): megaloblastic anemia + neurological symptoms, hypersegmented neutrophils C (ascorbic acid): scurvy (↑ bleeding, anemia, loose teeth) D: rickets (children), osteomalacia, tetany E: anemia, peripheral neuropathy, ataxia K: ↑ bleeding, ↑ PT/INR, ↑ PTT if severe Folate: megaloblastic anemia, sensory neuropathy

A 55-year-old man just had his first colonoscopy and uncomplicated polypectomy for 1-2 small (<10 mm) tubular adenomas. He has no family history of cancer. In the majority of patients with 1-2 small adenomas, what is the time frame recommended for a repeat colonoscopy after an initial colonoscopy and polypectomy is performed? A) 1 year to 3 years B) 3 years to 5 years C) 5 years to 10 years D) 6 months to 1 year

Correct answer: 5 years to 10 years Explanation: Colonic polyps occur in 30-50% of adults. Risk factors include age, genetic predisposition, a diet high in fat, red meat and low in fiber, tobacco use and obesity. There are three main types of polyps: hyperplastic (no risk of malignant transformation), adenomatous (most common; possible risk of malignant transformation) and malignant (already cancerous). Polyps may cause rectal bleeding (frank or occult), but typically, they are asymptomatic. They are commonly detected on sigmoidoscopy or colonoscopy, and usually removed if found. Microscopic examination is then performed to evaluate for the presence of malignant cells. Polypectomy carries the complication risks of bleeding and perforation. After polypectomy, for 1-2 small (<10 mm) tubular adenomas, repeat colonoscopy is recommended within 5 to 10 years. According to recommendations from the U.S. Multi-Society Task Force on colorectal cancer updated in 2020, repeat colonoscopy after

A 13-year-old boy with no significant medical history is referred for a colonoscopy after two of his siblings were found to have hundreds of adenomatous polyps on colonoscopy. Vital signs today include a HR of 75 bpm, BP of 120/80 mm Hg, RR of 20/min, oxygen saturation of 99% on room air, and T of 98.6°F. Physical examination reveals a regular rate and rhythm, lungs that are clear to auscultation, and a soft and non-tender abdomen. The patient's colonoscopy shows a similar finding with hundreds of adenomatous polyps. What is the risk of colorectal cancer in patients with this condition who are left untreated? A) 100% B) 20% C) 5% D) 50% E) 80%

Correct answer: A) 100% Explanation: The colonoscopy findings of the patient and his siblings are highly suspicious for classic familial adenomatous polyposis. Familial adenomatous polyposis is an autosomal dominant syndrome that is caused by germline mutations in the adenomatous polyposis coli gene. Familial adenomatous polyposis is classified as attenuated (11 to 99 adenomas) or classic (at least 100 adenomas) based on the number of colorectal adenomas identified. The polyps typically begin between 11-29 years of age, and colorectal cancer occurs in nearly 100% of patients who are not treated. The characteristic clinical manifestations of familial adenomatous polyposis include colonic polyps, fundic gland polyps, and duodenal adenomas. Patients also have an increased risk of other malignancies, such as follicular or papillary thyroid cancer, childhood hepatoblastoma, and central nervous system tumors. The diagnosis should be suspected in patients who are found to have more than 10 adenomas on colonoscopy. However, the diagnosis is confirmed by genetic testing confirming a germline mutation in the adenomatous polyposis coli gene. Patients who are suspected of having classic familial adenomatous polyposis should undergo flexible sigmoidoscopy or colonoscopy annually starting around 10-12 years of age. A full colonoscopy should be performed if colorectal adenomas are detected. Patients with classic familial adenomatous polyposis require colectomy at some point in the disease treatment. The indications include documented or suspected colorectal cancer, adenoma with high-grade dysplasia, significant symptoms related to colonic neoplasia (e.g., gastrointestinal bleeding), marked increases in polyp number on consecutive colonoscopy examinations, and an inability to survey the colon due to the extent of the polyps. - Esophagogastroduodenoscopy and thyroid ultrasonography. (routinely performed in patients with FAP)

A student health volunteer presents for evaluation after an accidental poke with a contaminated needle stick. You review the labs of the source patient which are as follows: Anti-HAV negative; HBsAg positive; HBsAb negative; anti-HBc IgM positive, Anti-HCV negative. Which of the following is the most appropriate explanation of these labs? A) Active infection with hepatitis B B) Previous exposure to hepatitis A C) Previous infection with both hepatitis B and C D) Vaccination against hepatitis B

Correct answer: Active infection with Hepatitis B Explanation: The source patient is HBsAg positive (the surface antigen) and HBsAb negative indicating an active hepatitis B infection. The presence of surface antigen (HBsAg) indicates that the patient has an active infection with hepatitis B, either acute or chronic. The presence of anti-HBc IgM indicates that the infection is acute.

A 12-month-old girl is brought in for evaluation of abdominal pain. She has been intermittently lying on the floor and "doubling up" with abdominal pain. The episodes last about one minute and then resolve. She had one episode of emesis. Physical exam reveals a palpable mass in the right lower quadrant of the abdomen. Which of the following is the most appropriate diagnostic test? A) Air/contrast enema B) Colonoscopy C) Plain abdominal radiographs D) Upper GI series with contrast

Correct answer: A) Air/contrast enema Explanation: An air/contrast enema is indicated when intussusception is suspected. Intussusception is a telescoping of the bowel into an adjacent segment and is the most common cause of intestinal obstruction in young children. This typically affects children between the ages of six months to three years of age and is characterized by bouts of intermittent, severe abdominal pain. It can be accompanied by vomiting and "currant jelly" stool that is the result of blood from venous infarction of the bowel along with mucous production. The most common area for this to occur is at the terminal ileum into the right colon. A barium enema can be both diagnostic and therapeutic, reducing the intussusception. Several attempts at reduction can be made through using this modality before surgery is indicated. Surgical resection is indicated if the bowel is gangrenous or cannot be reduced.

A 43-year-old man presents to the clinic for evaluation of perirectal pain that began last week. He states the pain has progressively worsened and has recently become so severe he cannot sit for more than a few minutes at a time. His medical history includes hypertension, diabetes, and hyperlipidemia, for which he takes metoprolol, enalapril, metformin, and simvastatin. Vital signs include HR 103 bpm, RR 17 breaths/min, BP 141/88 mm Hg, T 100.9°F, and SpO2 99% on room air. Physical examination reveals an erythematous patch of induration at the 2 o'clock perianal position. Primary intervention for this condition is undertaken with no complications. Which of the following is the best next step in the management of this patient's condition? A) Amoxicillin-clavulanate B) Colonoscopy C) Computed tomography of the abdomen and pelvis D) Methylprednisolone E) No further management is required

Correct answer: A) Amoxicillin-Clavulanate Explanation: An anorectal abscess is a collection of pus that most often occurs secondary to an infection from an obstructed anal crypt gland. While a perianal abscess is a simple anorectal abscess, perirectal abscesses involve multiple planes of the anorectum and are further classified based on anatomic location (i.e., ischiorectal, intersphincteric, supralevator, horseshoe). Patients with an anorectal abscess report severe pain in the anal or rectal area that is not associated with bowel movements. Due to infection, constitutional symptoms such as fever and malaise are often present. A superficial anorectal abscess is identified on physical examination as a localized area of fluctuance or a patch of erythematous, indurated skin. Purulent drainage may be present if the abscess has spontaneously opened. Deeper abscesses may have no obvious physical examination findings and require further investigation via digital rectal examination or diagnostic imaging, such as computed tomography, magnetic resonance imaging, or transperitoneal or endorectal ultrasound. Prompt surgical drainage is required for all perianal and perirectal abscesses. Anorectal abscesses that are undrained can continue to propagate and may progress to generalized systemic infection. Surgical skin incisions should be made close to the anal verge to reduce the likelihood of fistula formation. Following drainage, clinicians often pack the wound with sterile gauze, but there has been no proven benefit to this practice. Patients who undergo incision and drainage of an anorectal abscess should be prescribed a 4- to 5-day course of amoxicillin-clavulanate or ciprofloxacin in combination with metronidazole if they have a history of immunosuppression, diabetes, valvular heart disease, signs of systemic infection, or extensive cellulitis. Targeted antibiotic therapy can be achieved via wound culture and is appropriate for patients who have had multiple courses of antibiotics, have pain out of proportion to physical exam findings, are immunocompromised, or are at risk for methicillin-resistant Staphylococcus aureus infection. Antibiotic treatment decreases the risk of complications, such as systemic infection

A 54-year-old man presents complaining of epigastric pain that started several hours ago. The pain is moderate and sharp but does not radiate. He has a history of hypertension and an exploratory laparotomy 20 years ago. His vital signs on presentation include T 37°C, HR 95 bpm, and BP 136/80 mm Hg. His exam reveals a firm, mildly tender protruding mass in the epigastric region with no overlying skin changes. Which of the following is the best next step? A) Apply gentle steady pressure to the

Correct answer: A) Apply gentle steady pressure to the mass Explanation: This patient is presenting with a ventral hernia through the incision site from his exploratory laparotomy. Incisional hernias account for up to 20% of all abdominal wall hernias. They are often the result of excess wall tension or inadequate wound healing. Risk factors for the development of incisional hernias include obesity, age, wound infection, and certain medical conditions that increase the intra-abdominal pressure. All hernias fall into one of three categories: reducible, incarcerated, or strangulated. Reducible hernias are soft and easy to replace through the hernia defect. Incarcerated hernias are firm, often painful, and non-reducible by direct manual pressure. Strangulation occurs as a consequence of incarceration and results in impaired blood flow, leading to ischemia, necrosis, and obstruction. Skin changes overlying the hernia site may be seen, and patients are often toxic in appearance. A strangu

A 29-year-old pregnant patient presents with a three day history of diarrhea following a trip to Mexico. The patient describes having seven to eight loose, watery stools per day. She does not note any blood in the diarrhea and is otherwise asymptomatic. Vital signs are BP 116/72, HR 86, RR 15, T 38.7°C. Physical exam reveals mild dehydration, but is otherwise normal. After providing fluid replacement, what is the indicated treatment? A) Azithromycin B) Ciprofloxacin C) Symptomatic therapy with loperamide D) Trimethoprim-Sulfamethoxazole

Correct answer: A) Azithromycin Explanation: Traveler's diarrhea is the most common illness affecting travelers to low-income parts of the world. Common causes include Enterobacteriaceae, such as E. coli, Shigella spp, and Campylobacter, as well as norovirus and rotavirus. Parasitic infection, such as Giardia or Cryptosporidium, should also be considered, especially in the event of persistent (>14 d) or refractory diarrhea. The syndrome begins abruptly with several loose or watery stools per day and may include symptoms such as nausea, vomiting, and abdominal cramping. When patients present with gross blood mixed with stools or fever, the diarrhea has evolved into dysentery, indicating invasion of the intestinal mucosa. In the situation of diarrhea with fever or bloody stools, a stool culture should be performed. Fluid replacement is always the primary treatment of traveler's diarrhea. Most cases are self-limited and resolve on their own within three to five days of treatment with fluid replacement only. Antimicrobial therapy shortens the disease duration to about one day. Antibiotics are warranted to treat diarrhea in those who develop severe diarrhea, characterized by more than four unformed stools daily, fever, or blood, pus, or mucus in the stool. In addition, some travelers desire antibiotic treatment for milder disease if the illness is a large burden on a business trip or vacation. Azithromycin is the preferred antibiotic therapy in pregnant women and children. It is also the first choice for use in South and Southeast Asia where Campylobacter is common and has high resistance to fluoroquinolones. -Ciprofloxacin (B) is the antibiotic of choice for most destinations and is effective against most mucosally invasive pathogens, but is contraindicated in children and pregnancy. Antimotility agents such as loperamide (C) can provide symptomatic relief, but should not be used without antibiotics in patients with fever or bloody stools. Trimethoprim-sulfamethoxazole (D) is no longer a recommended treatment for traveler's diarrhea due to widespread antibiotic resistance.

A man presents with difficulty walking and painful calves. He has a history of alcohol use disorder and ulcerative colitis, both untreated. He reports no history of chronic diarrhea. A blood tox screen is negative for alcohol and illicit drugs. His examination is significant for several neurologic findings: altered proprioception, distal numbness, hyporeflexia, confusion, and nystagmus. Perioral and tongue examination is normal. These symptoms are most likely related to a deficiency of which of

Correct answer: A) B1 Explanation: Vitamin B1, thiamine, is necessary for energy, carbohydrate, lipid, and amino acid metabolism. It acts as a component of the coenzyme responsible for conversion of pyruvate to acetyl-CoA. Thiamine is also essential for proper nerve and muscle function. Risk factors include a diet high in refined sugars, alcohol use disorder, malnourishment, pancreatitis, liver disease, inflammatory bowel disease, bariatric surgery, renal failure and kidney dialysis, chemotherapy, cancer, AIDS, hyperthyroidism, and chronic diarrhea. Although uncommon in most resource-rich countries, thiamine deficiency still exists in present-day prisons. Chronic thiamine deficiency can lead to beriberi, a condition marked by cardiac ("wet" beriberi) and nervous system ("dry" beriberi) abnormalities. Concerning dry beriberi symptoms include numbness, paresthesias, decreased proprioception, ataxia, hyporeflexia, painful calves, confusion, abnormal eye movements, nystagmus, vom

A 54-year-old man with a history of gastroesophageal reflux disease and hypertension presents to the office for a new patient physical exam. He currently takes omeprazole 20 mg twice daily and lisinopril 10 mg once daily. He has taken omeprazole for 5 years but states he has struggled with heartburn for almost 15 years. He also reports a 20 pack-year smoking history. His vital signs are heart rate 70 bpm, blood pressure 125/85 mm Hg, respirations 20/min, oxygen saturation 98% on room air. His ph

Correct answer: A) Columnar-lined surface epithelium with goblet cells Explanation: Barrett esophagus is a condition that results from chronic gastroesophageal reflux disease (GERD). Chronic GERD changes the cellular makeup of the distal esophagus from stratified squamous epithelium to columnar-lined epithelium with goblet cells and is diagnosed via endoscopy with biopsy. The goblet cells represent intestinal metaplasia and are not typically seen in the distal esophagus. Risk factors for Barrett esophagus include male sex, age > 50 years old, GERD, central obesity, family history, and smoking. Screening for Barrett esophagus is indicated in men with two or more risk factors. It is important to diagnose the condition as it increases the risk of developing esophageal adenocarcinoma by up to 30%. There are no specific symptoms for Barrett esophagus, but patients will have prolonged symptoms of reflux. Treatment includes a high-dose proton pump inhibitor daily to reduce the risk of esoph

A patient presents with new onset odynophagia. Chest imaging reveals a lower esophageal stricture but rules out the possibility of neoplasia. Endoscopy shows mucosal inflammation and fibrosis. Which of the following is the most appropriate initial treatment? A) Dilation B) Diltiazem C) Famotidine D) Lactulose

Correct answer: A) Dilation Explanation: Esophageal strictures stem from three general causes: intrinsic esophageal disease that narrows the lumen (neoplasia, inflammation, fibrosis), extrinsic disease which constricts the lumen (neoplasia, lymph node enlargement), and altered peristalsis or lower esophageal sphincter function (dysmotility, smooth muscle, or nerve disease). The most common symptom is progressive solid dysphagia, while others include liquid dysphagia, heartburn, odynophagia, weight loss, chest pain, cough, asthma, food aspiration, and food regurgitation. These symptoms are also common in achalasia, esophageal motility disorders, and esophagitis. Esophageal strictures can be identified and evaluated with computed tomography, endoscopy, endoscopic ultrasound, and manometry. The current mainstay of treatment is dilation. Some sources encourage proton pump inhibitor use after dilation to help prevent recurrence.

Which of the following represents appropriate management of a thrombosed external hemorrhoid in the acute 48-hour setting? A) Excision B) Reduction C) Sitz baths D) Surgery referral

Correct answer: A) Excision Explanation: Thrombosed external hemorrhoids presenting within 48 - 72 hours of symptom onset should be excised. Hemorrhoids occur when small vascular beds become engorged. Both external and internal hemorrhoids can present with the complaint of bright red blood per rectum. Nonthrombosed external hemorrhoids can be managed with warm water baths, analgesics, stool softeners, and high-fiber diet. Acutely thrombosed external hemorrhoids, however, will not improve with conservative therapy and require excision. Incision of the hemorrhoid is followed by complete evacuation of the clot. Failure to fully remove the clot results in rebleeding, swelling, and skin tag formation. If the thrombosed external hemorrhoid is not excised, it will go on to ulcerate over the next several days.

Which of the following combinations of hepatitis B serologic markers is indicative of hepatitis B immunity secondary to vaccination? A) HBsAg negative, anti-HBc negative, anti-HBs positive B) HBsAg negative, anti-HBc positive, anti-HBs positive C) HBsAg positive, anti-HBc positive, IgM anti-HBc negative, anti-HBs negative D) HBsAg positive, anti-HBc positive, IgM anti-HBc positive, anti-HBs negative

Correct answer: A) HBsAg negative, anti-HBc negative, anti-HBs positive Explanation: Hepatitis B surface antigen (HBsAg) negative, hepatitis B core antibody (anti-HBc) negative, hepatitis B surface antibody (anti-HBs) positive is the serologic marker combination indicative of hepatitis B immunity due to the administration of the hepatitis B vaccination. HBsAg is a marker for the hepatitis B surface antigen, a protein on the surface of the hepatitis B virus. The presence of this protein in the body conveys that the person is currently infected and that they have either acute or chronic hepatitis B. Anti-HBc is the total hepatitis B core antibody that appears at the onset of symptoms and remains for life. This marker indicates a previous or ongoing infection with hepatitis B. Anti-HBs is the hepatitis B surface antibody, which is positive when a patient has recovered from the disease or when a patient has been immunized against the disease either by vaccination or prior infection. When a patient has immunity due to vaccination, the HBsAg will be negative since they are not acutely or chronically infected, the anti-HBc will be negative as they have not had a previous or active infection, and the anti-HBs will be positive because they were immunized against the virus by vaccination.

A 55-year-old man with a history of alcohol use disorder and cirrhosis presents to the emergency department reporting abdominal distension and weight gain. He states that his symptoms have been slowly progressing over the past 3 weeks. He is not taking any medications and reports no orthopnea, abdominal pain, lethargy, or confusion. Vitals are temperature 37°C, blood pressure 120/80 mm Hg, pulse 89 bpm, and SpO2 98% on room air. Physical exam reveals a man with jaundice in no acute distress. His abdomen is markedly distended and nontender to palpation in all four quadrants with bulging flanks. There is dullness to percussion in the flank region. The remainder of the physical exam is unremarkable. Which of the following is the best next step in management? A) Obtain abdominal ultrasound B) Perform diagnostic paracentesis C) Prescribe oral diuretic with outpatient follow-up in 2 weeks D) Prescribe oral lactulose with outpatient follow-up in 2 weeks E) Recommend sodium restricted diet

Correct answer: A) Obtain an abdominal ultrasound Explanation: This patient is presenting with ascites, or the accumulation of fluid within the peritoneal cavity. The most common cause of ascites in the United States is cirrhosis, which accounts for about 80% of cases. Other common causes include infection, malignancy, and heart failure. The development of ascites in patients with cirrhosis is related to portal hypertension, which causes vasodilatory substances such as nitric oxide to accumulate, resulting in splanchnic vasodilation and renal hypoperfusion. Hypoperfusion of the kidneys activates the renin-angiotensin-aldosterone system (RAAS), causing increased fluid retention. Patients with ascites typically present with abdominal distension that may be accompanied by abdominal discomfort, weight gain, shortness of breath, and peripheral edema. The distension is usually progressive and develops over the course of several weeks. If the ascites is secondary to cirrhosis, patients may also have signs of hepatic decompensation, such as confusion (encephalopathy) or evidence of gastrointestinal bleeding. Patients with ascites commonly have bulging flanks with dullness to percussion (flank dullness) on examination. They may also have shifting dullness (change in location of dullness to percussion when the patient is repositioned). If there are indications of ascites based on history and physical exam, the first step in diagnosis should be abdominal ultrasound to confirm its presence and to look for evidence of cirrhosis or malignancy. Diagnostic paracentesis is most useful for determining the cause of ascites. Ascites fluid should be evaluated for appearance (e.g., clear, bloody, cloudy), serum-to-ascites albumin gradient, cell count and differential, and total protein concentration. Most occurrences of small amounts of cirrhosis-related ascites can be managed with dietary sodium restriction alone. For patients with moderate to large ascites, diuretic therapy, therapeutic paracentesis, or both may be necessary. Diuretic treatment is usually with spironolactone alone or in combination with furosemide. The prognosis for patients with ascites due to cirrhosis is poor, with a < 50% 2-year survival rate after the ons

A 45-year-old man presents to the clinic reporting retrosternal chest pain and a bitter, metallic taste in his mouth for the past several months. The symptoms occur about 2 to 3 times per week, about 30 minutes after meals, and sometimes cause him to wake up from sleep at night. His symptoms are relieved by taking antacids and standing up from a supine position. He has a body mass index of 34 kg/m2, a temperature of 98.4°F, blood pressure of 128/72 mm Hg, and a heart rate of 82 bpm. Which of the following is the primary mechanism of his underlying condition? A) Decreased lower esophageal sphincter tone B) Defective propagation of peristaltic waves through the esophageal wall C) Delayed gastric emptying D) Gastric outlet obstruction E) Increased gastric acid production

Correct answer: A) decreased lower esophageal sphincter tone Explanation: Gastroesophageal reflux disease (GERD) is a common condition in which the contents of the stomach flow back into the esophagus. The primary mechanism of this disease is caused by decreased tonicity of the lower esophageal sphincter (LES). The LES acts as a barrier to prevent reflux back into the esophagus, and inappropriate relaxation of the LES can lead to symptoms such as heartburn (retrosternal pain or burning) and regurgitation (reflux of sour or bitter gastric contents into the mouth) that typically occur up to 3 hours after eating. Other factors that contribute to symptoms include decreased esophageal motility, gastric outlet obstruction, the presence of a hiatal hernia, and truncal obesity. Dietary and lifestyle factors, such as alcohol and tobacco use, chocolate, ingestion of high-fat foods, and coffee, are also contributors. Although heartburn and regurgitation are the most common symptoms, patients may also report dyspepsia, dysphagia, belching, chest pain (which can mimic cardiac chest pain), cough, hoarseness, sore throat, and increased salivation. Symptoms are typically exacerbated by eating large meals or by lying down after meals. In uncomplicated disease, a physical exam and laboratory findings are unremarkable. Patients are typically diagnosed by taking a thorough history and treated empirically with H2-receptor antagonists or a proton pump inhibitor for 4-8 weeks. Patients with persistent or more worrisome symptoms, such as weight loss, iron deficiency anemia, and painful swallowing, may warrant further investigation to identify complications of GERD. Upper endoscopy with biopsy is helpful in documenting tissue damage in patients with gastroesophageal reflux and detecting other complications of GERD, such as erosive esophagitis, esophageal or peptic stricture, Barrett metaplasia, esophageal ulcers, and esophageal adenocarcinoma. An esophageal pH test and the combined esophageal pH-impedance test are the most sensitive and specific tests for GERD but are unnecessary in most patients with typical symptoms. Additional studies include barium esophagography, which can be used to identify the presence of a stricture, and

A 46-year-old man presents to the emergency department reporting abdominal pain, bloating, nausea, anorexia, and vomiting. His symptoms have been worsening since onset three days ago. His last bowel movement was four days ago, but he continues to pass flatus. Which of the following tests has the highest sensitivity for confirming the most likely diagnosis? A) Abdominal CT with contrast B) Abdominal ultrasound C) Abdominal X-ray D) Colonoscopy

Correct answer: Abdominal CT w contrast Explanation: Abdominal CT with contrast is most likely to confirm a small bowel obstruction as it has the highest sensitivity and specificity of the various imaging modalities listed. Additionally, a CT scan can identify complications, such as ischemia or perforation, localize the transition point, evaluate for malignancy, and rule out competing diagnoses, such as volvulus or intussusception. Often, an initial abdominal X-ray is obtained prior to CT evaluation and may show dilated loops of bowel with air-fluid levels or proximal dilation with a gasless distal bowel. Adjunctive tests include ultrasound, magnetic resonance enterography, and contrast X-rays. Symptoms of small bowel obstruction include nausea, vomiting, intermittent abdominal cramping, abdominal distension, constipation, and, if the obstruction is complete, obstipation with failure to pass flatus. Partial obstruction may be associated with hyperactive bowel sounds and high-pitched

A 57-year-old man presents with a change in stools over the last 4 months. His stools are intermittently bloody, and he has experienced a decline in his appetite associated with a 15-pound unintentional loss of weight. Which of the following is the most likely explanation for his symptoms? A) Adenocarcinoma of the colon B) Gastrointestinal carcinoid tumor C) Primary colorectal lymphoma D) Squamous cell carcinoma of the colon

Correct answer: Adenocarcinoma of the colon Explanation: Adenocarcinoma of the colon is the most common type of colorectal cancer and the third leading cause of death due to cancer in the United States. Symptoms may include abdominal pain, change in bowel habits, weight loss, loss of appetite and hematochezia. Diagnosis is usually accomplished by colonoscopy with biopsy, though CT colonography or barium enema may be considered, as well. CT or MRI may be used for help with staging. Other laboratory findings may include iron-deficiency anemia, elevated liver transaminase levels, and an elevated carcinoembryonic antigen level. Treatment includes resection of the cancer with regional lymph node dissection, as well as chemotherapy and radiation for patients with more advanced disease.

A 54-year-old man with cirrhosis presents for evaluation of abdominal pain. The pain is diffuse throughout the abdomen and associated with subjective fever at home. He has no vomiting, diarrhea or change in mental status. His vital signs are T 100.6°F, HR 98, BP 140/88, RR 12, and oxygen saturation of 100% on room air. Bedside ultrasound demonstrates ascites. Which of the following is an indication for intravenous antibiotics? A) Ascitic fluid neutrophil count of 300 cells/mcL B) Ascitic flu

Correct answer: Ascitic fluid neutrophil count of 300 cells/mcL Explanation: The patient's presentation is concerning for spontaneous bacterial peritonitis (SBP), an acute bacterial infection in the ascitic fluid of patients with ascites in the setting of liver disease. Most commonly, gram negative enteric organisms are responsible for the infection and the treatment of choice is an intravenous third generation cephalosporin. Diagnosis is made based on an ascitic neutrophil count > 250 cells.

Which of the following is the most effective initial treatment for a benign esophageal stricture? A) Balloon dilation B) Balloon dilation and proton pump inhibitor C) Esophageal stent D) Nissen fundoplication

Correct answer: B) Balloon dilation and PPI Explanation: Balloon dilation and proton pump inhibitor is the most effective initial treatment for a benign esophageal stricture. An esophageal stricture is the narrowing of the esophagus caused by the buildup of acid and is one of the complications of gastrointestinal reflux disease. Symptoms include heartburn, dysphagia, coughing, and shortness of breath. Treatment is aimed toward decreasing the symptoms of dysphagia and preventing the stricture from reoccurring. Therefore, the most effective initial treatment is to dilate the stricture using either a balloon dilator or a mechanical dilator followed by prescribing a proton pump inhibitor to prevent reoccurrence of acid buildup in the esophagus. Administering a proton pump inhibitor does not decrease the need to achieve adequate dilation with the balloon, but it has been proven to help reduce the need for subsequent dilations.

A 23-year-old man presents with rectal pain. He also reports yellow discharge and tenesmus. He is sexually active with men and has receptive anal intercourse. Which of the following is the most appropriate treatment? A) Ceftriaxone 1 gm IV and azithromycin 1 gm PO B) Ceftriaxone 500 mg IM and doxycycline 100 mg PO BID for 7 days C) Ciprofloxacin 500 mg PO BID for 7 days and metronidazole 500 mg PO TID for 7 days D) Metronidazole 500 mg PO TID for 7 days

Correct answer: B) Ceftriaxone 500mg IM and Doxycycline 100 mg PO BID 7 days. Explanation: This patient is presenting with findings consistent with proctitis. Sexually transmitted infections of the anorectal area are an important consideration in men who have sex with men or women who take part in anal intercourse. In this case, receptive anal intercourse puts him at risk for sexually transmitted organisms as the cause of his proctitis, particularly N. gonnorhoeae and C. trachomatis. The incubation period after an exposure is typically 5 to 7 days. Patients develop anal pruritus, tenesmus, and discharge which may be bloody or purulent. The appropriate treatment is ceftriaxone 500 mg IM (1 g for patients > 150 kg) for coverage of gonorrhea and doxycycline 100 mg PO BID for 7 days for coverage of chlamydia. Herpes simplex virus is another important cause of proctitis and careful examination of the anorectal area for herpetic lesions is indicated.

A 37-year-old obese woman presents with symptoms of biliary colic for the past 3 months. An ultrasound is negative for any gallstones. Which of the following is the most appropriate next step for this patient? A) Cholecystectomy B) Cholecystokinin-HIDA scan C) CT scan D) Endoscopic Retrograde Cholangiopancreatography (ERCP)

Correct answer: B) Cholecystokinin-HIDA Scan Explanation: For patients presenting with symptoms highly suggestive of gallstones but without gallstones on imaging, a cholecystokinin-HIDA scan should be performed. Cholecystokinin-HIDA scan, is a variant of the hepatobiliary iminodiacetic (HIDA) scan and can be used in the elective setting to assess gallbladder contractility and calculate an ejection fraction. In up to 20% of the patients with symptoms typical of biliary colic, no gallstones are seen on imaging, possibly because of small size or stone composition. Cholelithiasis is one of the most common and costly of all the gastrointestinal diseases. The incidence of gallstones increases with age. At-risk populations include a history of diabetes mellitus, obesity, women, rapid weight cyclers, and patients on hormone therapy or taking oral contraceptives. Most patients with gallstones are asymptomatic and gallstones are discovered incidentally during ultrasonography or other imaging o

Over the last 7 months, a 13-year-old girl has had intermittent abdominal pain, which has made her quite irritable. The abdominal pain is associated with arthralgias and general malaise. Review of systems reveals that she has lost 5 kg (11 lb) and has painful bowel movements. Which one of the following is the most likely cause of these symptoms? A) Celiac disease (gluten enteropathy) B) Crohn's disease C) Irritable bowel syndrome D) Ulcerative colitis

Correct answer: B) Chron's disease Explanation: Crohn's disease is the most common chronic inflammatory bowel disease which occurs during adolescence and young adulthood, with a second peak at 50-80 years of age. The manifestations of Crohn's disease are dependent on the site of involvement, but systemic signs and symptoms are more common than with ulcerative colitis. Crohn's disease presents with chronic diarrhea, crampy abdominal pain, fever, weight loss, and fatigue. Perianal disease (anal fissures, perirectal abscesses, and anorectal fistulas) is also common in Crohn's disease. Crohn's disease may affect any part of the GI tract, from the mouth to the anus, but typically affects the small bowel and colon. Irritable colon and celiac disease may mimic symptoms of Crohn's disease, but objective findings of weight loss and anal lesions are extremely uncommon.

Which of the following is a risk factor associated with the development of pancreatic cancer? A) Alcohol consumption B) Cigarette smoking C) Diet high in red meats D) Sedentary lifestyle

Correct answer: B) Cigarette smoking Explanation: Cigarette smoking has been associated with 20 to 25 percent of all pancreatic cancers. It is the most common environmental and preventable risk factor associated with the disease. Other risk factors include genetic predisposition, chronic pancreatitis, and longstanding diabetes. Pancreatic cancer is the fourth leading cause of death from cancers in the United States and is often discovered at the time of metastatic disease. The five-year survival rate is just 6 percent for all stages. The majority of cases are caused by infiltrating ductal carcinomas affecting individuals between the ages of 65 and 84 most frequently. Physical exam findings may show jaundice, cachexia, or left supraclavicular lymphadenopathy (Virchow's node). Diagnosis may be confirmed with a EUS-guided fine needle aspiration. Treatment includes tumor resection when possible and adjuvant chemotherapy.

A 52-year-old man with a history of cirrhosis presents with worsening confusion. His spouse reports he has been taking his lactulose at home. Which of the following is a common cause of worsening hepatic encephalopathy in a patient such as this? A) Cardiac ischemia B) Constipation C) Hyperkalemia D) Pancreatitis

Correct answer: B) Constipation Explanation: Hepatic encephalopathy is a state of confusion caused by acute and chronic liver disease. The pathophysiology involves many pathways but is directly related to the impaired liver's inability to perform its regular metabolic function. Ammonia is used as a marker of the process. Ammonia that is absorbed in the GI tract is metabolized to urea by the liver for excretion. When this does not occur, ammonia levels increase and ultimately cross the blood-brain barrier leading to cerebral dysfunction. Hepatic encephalopathy covers a spectrum of disease from mild cognitive impairment to coma. On physical examination, asterixis is characteristic of the process (flapping of the wrist when it is extended). There are many causes of acutely worsening hepatic encephalopathy including constipation due to an increase in intestinal ammonia production and subsequent absorption. Other causes include kidney failure, gastrointestinal bleeding, infection, medica

A 35-year-old man with a history of gastroesophageal reflux disease presents to the emergency department with chest pain and fever. An outpatient esophagogastroduodenoscopy was performed 3 days ago and revealed no gastric ulcers. On exam, he is ill-appearing and tachycardic. Which of the following is the best next step in management? A) Arrange for repeat emergent esophagogastroduodenoscopy B) Consult thoracic surgery C) Order a barium esophagogram D) Place a nasogastric tube

Correct answer: B) Consult thoracic surgery Explanation: This patient presents with esophageal perforation, and an emergent thoracic surgery consult is indicated. Esophageal perforation can be caused by foreign body ingestion, trauma, forceful emesis (Boerhaave syndrome), or iatrogenesis (e.g., esophagogastroduodenoscopy). The majority of esophageal perforations are iatrogenic due to instrumentation of the esophagus. The patient in this question had a recent esophagogastroduodenoscopy (EGD) performed. Esophageal perforation is associated with high morbidity and mortality. The lack of a serosal covering means that perforation at any level will lead to direct spillage into the mediastinum with subsequent mediastinitis. Rupture due to Boerhaave syndrome typically occurs in the left posterior distal esophagus, an area of intrinsic weakness. Rupture due to foreign body or instrumentation classically occurs at one of the three areas of anatomic narrowing (the cricopharyngeal muscle, the cr

A 3-year-old boy presents after swallowing a quarter. He now feels it is stuck. At which location is an obstruction most likely to occur? A) Aortic arch B) Cricopharyngeus muscle C) Gastroesophageal junction D) Tracheal bifurcation

Correct answer: B) Cricopharyngeus muscle Explanation: Swallowed foreign bodies in pediatrics most commonly obstruct at the level of C6, the Cricopharyngeus muscle, one of the four natural areas of luminal narrowing in the esophagus. The Cricopharyngeus is the muscle of the upper esophageal sphincter (UES). Depending on the type of material, a plain radiograph may not reveal the object, particularly with food bolus obstructions. Pediatric patients often swallow objects like coins, and it may be difficult to ascertain whether a coin is in the trachea or esophagus. On a soft tissue X-ray, a coin located in the esophagus appears in the coronal plane, "en face," as opposed to appearing sagitally when in the trachea. Most foreign bodies < 2 cm x 5 cm will pass spontaneously through the GI tract. If the patient is without severe symptoms, foreign bodies less than this size are observed for 24 hours, as only 10-20% require endoscopic removal. Fewer than 1% will require surgical removal. Pediatric patients account for > 75% of esophageal foreign bodies. Other at-risk patients include edentulous persons, those with psychiatric diseases, prisoners, and those with underlying structural abnormalities of the esophagus (rings, strictures, diverticula, malignancy). -Aortic arch (A) at the level of T4 is the second most common location for foreign body obstruction in the esophagus. The gastroesophageal junction (C) at T11 is the third most common location for obstruction. The tracheal bifurcation (D) at T6 is the fourth most common site, particularly as the left mainstem bronchus pushes against the esophagus. The thoracic inlet at T1 is the least common location for obstruction.

Of the following, which is the most common infectious etiology of diarrhea in patients with AIDS? A) Candida B) Cryptosporidium C) Cytomegalovirus D) Mycobacterium avium complex

Correct answer: B) Cryptosporidium Explanation: As the CD4 count drops, patients with AIDS are at risk for opportunistic infections of the gastrointestinal tract. Worldwide, diarrhea is a source of significant morbidity and mortality of HIV-positive patients because of poor access to antiretroviral therapy. HIV infects and damages the gut-associated lymphoid tissue (GALT) which leads to frequent infection in those with poorly controlled disease and lower CD4 counts. HIV medications also cause diarrhea as a side effect. Additionally, patients with longstanding disease develop malabsorption syndromes from the chronic inflammatory changes of the intestinal tract. In patients with AIDS, Cryptosporidium is a common cause of diarrhea when the CD4 count falls below 100 cells/mm3.

A 52-year-old man with a history of obesity presents to your office with complaints of burning chest pain, food regurgitation and cough approximately once every week. He tells you that he likes to eat spicy foods and often eats a large meal shortly before bedtime to help him sleep. Which of the following is the most appropriate initial therapy? A) Bismuth subsalicylate B) Famotidine C) Metoclopramide D) Omeprazole

Correct answer: B) Famotidine Explanation: Gastroesophageal reflux disease (GERD) occurs when excessive amounts of gastric juice reflux into the esophagus causing uncomfortable symptoms or complications. Patients with GERD present with esophageal symptoms including heartburn, dysphagia and regurgitation. Extraesophageal symptoms such as cough, sore throat, hoarseness, noncardiac chest pain, and erosion of the teeth enamel may also be seen. Patients with overweight or obesity have a greater risk of developing GERD. The diagnosis of GERD may be made clinically and the role of endoscopy is controversial. Upper endoscopy is recommended when the diagnosis is unclear and in patients with alarm features including recurrent vomiting, gastrointestinal bleeding, anemia, weight loss and dysphagia. Other indications for endoscopy include GERD that is refractory to initial treatment, men older than 50 years with risk factors for Barrett esophagus and esophageal carcinoma, and patients with severe erosive esophagitis. Treatment strategies involve using either a step-up or step-down approach to therapy, and all include lifestyle modifications as an initial recommendation. Step-up therapy is recommended for patients with mild GERD, which is defined as episodes occurring less than twice every week. Initial treatment is with a histamine-2 receptor antagonist (H2 blocker), such as famotidine. Although H2 blockers are not as effective as proton pump inhibitors in the treatment of GERD, H2 blockers are less associated with adverse effects and thus may be appropriate in patients with milder symptoms.

An 11-year-old girl is seen in the clinic for a well-child visit and is found to have a BMI in the 96th percentile for age and sex. What additional studies should be ordered, according to the American Academy of Pediatrics? A) Abdominal ultrasound to look for fatty liver disease B) Fasting lipid panel, fasting glucose, alanine aminotransferase C) Hemoglobin A1C and complete metabolic panel D) No additional studies are recommended until age 12 years

Correct answer: B) Fasting lipid panel, fasting glucose, and alanine aminotransferase Explanation: According to the American Academy of Pediatrics (AAP) guidelines published in January 2023, children starting at age 10 years with a BMI ≥ 95th percentile are recommended to have a fasting lipid panel, alanine aminotransferase and fasting glucose to screen for dyslipidemia, prediabetes, and nonalcoholic fatty liver disease (NAFLD). This should be done every 2 years. BMI is used to screen for obesity and is calculated from the weight in kilograms divided by the square of the height in meters. BMI can be plotted on an age- and gender-specific graph. In children and adolescents, overweight is defined as a BMI ≥ 85th to 94th percentile, while obesity is a BMI ≥ 95th percentile. Pediatric patients with obesity have a higher risk of type 2 diabetes mellitus, asthma, NAFLD, cardiovascular disease, and anesthesia complications. They also experience higher rates of depression, low self-es

A woman presents with significant family and work related stress. She has been drinking alcohol more frequently than usual. For the past two weeks, she has noticed upper central abdominal pain, nausea, and loss of appetite. Initial laboratory testing reveals a mild anemia. You decide to order an esophagogastroduodenoscopy, which only reveals gastric epithelial inflammation. Which of the following is the most likely diagnosis? A) Esophagitis B) Gastritis C) Pancreatitis D) Peptic ulcer disease

Correct answer: B) Gastritis Explanation: Gastritis refers to inflammation of the stomach epithelium secondary to an inflammatory response. The most common cause of acute gastritis is due to H. pylori infection. There is poor correlation between symptoms (pain and dyspepsia) and endoscopic findings. However, acute H. pylori gastritis can cause sudden onset of epigastric pain, nausea, and vomiting, while gastropathy caused by NSAID use, alcohol, or portal hypertension can present with GI bleeding. Exam is usually unremarkable. Diagnosis is mainly clinical but can include noninvasive H. pylori testing, evaluation for suspected autoimmune gastritis via antiparietal cell and intrinsic factor antibodies, and analysis of the serum pepsinogen I-to-II ratio. Endoscopy with biopsy can provide a pathologic diagnosis. Treatment generally consists of avoidance of causative factors (e.g. NSAIDs, alcohol) along with prophylaxis with proton-pump inhibitors. Which hematologic abnormality is associated with gastritis? Answer: Pernicious anemia (gastric parietal cell destruction -> decreased intrinsic factor -> less vitamin B12 to produce RBCs).

A 35-year-old man presents to the primary care clinic with heartburn after eating. Pain occurs mostly in the evening, approximately 30 minutes after consuming his evening meal. He also reports mild regurgitation and belching at night. He reports no dysphagia, bloody stools, or weight loss. He had an EGD 2 months ago and was found to have gastritis and negative for H. pylori. For the past 2 months, he has taken omeprazole 20 mg every morning but is on no other medication, and he has no other sign

Correct answer: B) Increase omeprazole to BID. Explanation: Gastroesophageal reflux disease (GERD) is a common condition where reflux of stomach contents produces symptoms of heartburn and dyspepsia. Other symptoms include regurgitation, burning chest pain, cough, or burping. Relaxation of the lower esophageal sphincter allows reflux into the esophagus, and GERD symptoms are usually worse after meals or when reclining or laying down. Uncomplicated GERD is a clinical diagnosis and usually does not require any further diagnostics or laboratory tests. The goal of treatment is to prevent complications and provide patients with relief from their symptoms. Initially, medication therapy is a once-daily proton pump inhibitor (PPI) given 30 minutes before the morning meal. H2 receptor antagonists are sometimes used but are not as effective as PPIs. Lifestyle modifications, such as eating smaller meals and not reclining after eating, should also be discussed with patients diagnosed with GERD. I

An 82-year-old man presents with acute symptoms of ischemic colitis. He subsequently undergoes emergent bowel resection surgery. The surgical report details an infarction of the descending colon and recto-sigmoid junction. Pathology in which of the following arteries is the most likely cause of this condition? A) Celiac trunk B) Inferior mesenteric artery C) Portal vein D) Superior mesenteric artery

Correct answer: B) Inferior mesenteric artery Explanation: Ischemic colitis frequently occurs in the elderly population, typically on the left side of the abdomen. Symptoms range from transient colitis and reversible colopathy to chronic ulcerating colitis, stricture, gangrene and fulminant colitis. Clinical manifestations include crampy left lower quadrant pain, frank or occult fecal blood, fever and peritoneal signs. The main etiologies are not embolic or thrombotic in nature, but are felt to be due to a change in total body circulatory status or anatomical and functional local changes in the mesenteric vessels. Many times, the exact etiology is unknown. The left "half" of the large intestine, sigmoid colon and rectum are supplied by the inferior mesenteric artery. The inferior mesenteric artery supplies the descending colon, sigmoid colon, and part of the rectum. It forms an anastomosis with the superior mesenteric artery.

A 45-year-old woman with a history of osteoarthritis, hypertension, hypercholesterolemia, and tobacco use presents to clinic. The patient states that she has been experiencing a burning epigastric pain that occurs several hours after eating a meal and is relieved by antacids or food. Several times, the pain has awakened her from sleep. She reports no nausea, vomiting, diarrhea, or bloody stools. Her current medications include lisinopril, rosuvastatin, and naproxen. Vitals are temperature 37°C,

Correct answer: B) Inhibition of cyclooxygenase 1- leading to prostaglandin deficiency Explanation: This patient presents with peptic ulcer disease (PUD). Ulcers, which can occur in the stomach, duodenum, or both, are defined as disruptions of mucosal integrity due to inflammation that extends through the muscularis mucosa. Patients with PUD commonly present with epigastric pain or discomfort, but some patients are asymptomatic. Associated symptoms include nausea, early satiety, and abdominal fullness. Depending on the location of the ulcers, the pain characteristics will vary. Patients with duodenal ulcers will experience pain several hours after a meal and at night when acid secretion occurs in the absence of a food buffer. The consumption of food or antacids typically improves symptoms. Alternatively, patients with gastric ulcers experience worsening of symptoms with the consumption of food. Based on these characteristics, the patient in the vignette is most likely to have duodena

Which of the following statements is most correct regarding appendicitis? A) An appendicolith is identified in the majority of cases B) Leukocytosis is seen in the majority of cases C) Perforation is rare in patients younger than 2 years of age D) The presence of an appetite makes the diagnosis unlikely

Correct answer: B) Leukocytosis is seen in majority of cases Explanation: Appendicitis is the most common surgical cause of abdominal pain. The highest incidence occurs in patients 10 to 30 years of age, and the highest misdiagnosis rate is in infants and the elderly due to atypical presentations. Leukocytosis (> 10,000/mcL) occurs in up to 80% of patients diagnosed with appendicitis, however a normal white blood cell count should not be used to rule out the diagnosis. The primary inciting event is obstruction of the appendiceal lumen, commonly from an appendicolith, lymphoid hyperplasia, or tumor. Obstruction leads to increased intraluminal pressure and distention of the appendix (visceral pain/periumbilical pain). Increased distention leads to vascular compromise of the appendiceal wall and bacterial invasion causing localized peritoneal inflammation (somatic pain/RLQ pain).

A 61-year-old man presents with loss of appetite, unintentional weight loss, and dyspepsia. Endoscopy shows a gastric mass at the curvature of the stomach, and he is diagnosed with gastric carcinoma. Which of the following is true about his condition? A) Chemotherapy and radiation are standard initial therapy for localized disease B) Men have a higher risk of developing this condition C) The 5-year survival rate of advanced disease is 80% D) White individuals are more likely to develop this co

Correct answer: B) Men have a higher risk of developing this condition Explanation: The most common type of gastric cancer is adenocarcinoma. Men develop this condition two times as often as women. Other risk factors for the development of gastric cancer include smoking, persistent Helicobacter pylori exposure, long-term ingestion of foods with nitrates, and the presence of certain types of gastric polyps, such as adenomatous and hyperplastic. Gastric cancer develops more commonly in Asian, Black, and Hispanic populations. Symptoms of this condition usually do not occur early in the disease. Patients may report abdominal pain, loss of appetite, nausea, difficulty swallowing, and unintentional weight loss as the tumor advances. Iron deficiency anemia and occult blood in the stool may be found. A gastric mass may be palpable in some patients. Endoscopy with cytology and biopsy are recommended for any patient over 50 years who requires ongoing acid suppression to control symptoms of dys

Which of the following is true regarding intestinal intussusception? A) Adults are more affected than children B) Most adult intussusception cases involve the small bowel C) Most children with intussusception have a pathologic lesion D) The classic triad of abdominal pain, mass, and heme-positive stools is usually seen in adults with intussusception

Correct answer: B) Most adult intussusception cases involve the small bowel Explanation: Intussusception in adults is rare but when present, involves the small bowel in 80% of cases. Intussusception involves telescoping of a proximal segment of the bowel into a distal segment. This process causes the mesenteric blood supply to the section of bowel to become compromised leading to ischemia, edema, infarction, and possibly gangrene. In adults, intussusception typically presents with symptoms consistent with partial bowel obstruction. Vomiting, bleeding per rectum, and constipation are common. The abdomen may become distended as well. As the disease progresses and bowel is compromised, sepsis may ensue. In adults, large bowel obstruction is frequently associated with malignancy. Although plain radiographs can be suggestive of the diagnosis and rule out perforation, US is most commonly used, CT is the definitive study for identification of the lead point.

Which vitamin deficiency presents with photosensitive dermatitis, diarrhea, and dementia? A) Cyanocobalamin B) Niacin C) Pyridoxine D) Thiamine

Correct answer: B) Niacin Niacin (B3) deficiency causes the clinical presentation of pellagra: dermatitis, diarrhea, and dementia. Niacin is a vitamin found in poultry, meat, and fish. Deficiency is rare in resource-rich countries but can be seen in patients with alcohol abuse disorder or other chronically malnourished states. Patients will present with raw skin and a photosensitive-pigmented dermatitis that is more pronounced in sun-exposed areas. Additionally, diarrhea and dementia are commonly seen. Niacin administration is the focus of treatment, although other concurrent vitamin deficiencies should be investigated.

A man who is pale and ill-appearing presents to the emergency department with reports of two episodes of bright red vomitus. A medical history review reveals alcohol use disorder, chronic hepatitis, and cirrhotic liver disease. Vitals include a HR of 105 bpm, RR of 18/min, BP of 96/55 mm Hg, T of 97.7°F, and SpO2 of 95% on room air. Laboratory testing reveals hemoglobin 10.1 g/dL, hematocrit 38.7%, INR 1.7, and platelet count 82,000/μL. Two large-bore intravenous access lines are placed with aggressive fluid repletion and prophylactic antibiotic administration initiated. What is the next step in treatment? A) Fresh frozen plasma transfusion B) Octreotide infusion C) Platelet transfusion D) Propranolol administration E) Transvenous intrahepatic portosystemic shunt

Correct answer: B) Octreotide infusion Explanation: Esophageal varices are dilated submucosal veins that develop secondary to portal hypertension. This vascular abnormality is present in 50% of patients with cirrhosis of the liver and may result in significant upper gastrointestinal bleeding. Patients with actively bleeding esophageal varices present with signs and symptoms of acute gastrointestinal hemorrhage, including hematemesis and melena. Acute gastrointestinal hemorrhage results in hypovolemia, postural vital signs, or shock in patients with esophageal varices. Laboratory testing should include a complete blood count, platelet count, prothrombin time, INR, serum liver enzymes, serum creatinine, and blood urea nitrogen. Varices that are actively bleeding require blood type and crossmatch to prepare for potential transfusion. Any patient with bleeding esophageal varices requires stabilization with intravenous fluid replacement. Antibiotic prophylaxis with intravenous agents such as ceftriaxone reduces in-hospital mortality and the risk of serious infection in patients with esophageal varices. Vasoactive substances (octreotide, somatostatin, terlipressin) reduce splanchnic and hepatic blood flow and portal venous pressures and should be administered after patients are initially stabilized. Additionally,l band ligation via endoscopy can improve survival. Vitamin K should be administered to patients with cirrhosis and an abnormal prothrombin time and patients with hepatic encephalopathy and increased ammonia should additionally be treated with lactulose. Endoscopy is performed within 12-24 hours of hemodynamic stabilization to exclude other or associated causes of upper gastrointestinal bleeding. Balloon tamponade can also be used to control acute bleeding while more definitive treatment measures are being prepared. Portal pressure can be reduced long-term with the administration of nonselective beta-blockers (e.g., propranolol, nadolol), with a target heart rate of 55-60 bpm. A transvenous intrahepatic portosystemic shunt (TIPS) procedure is appropriate for patients who have had recurrent bleeding from gastric varices, who have portal hypertensive gastropathy, who have not responded to endoscopy or p

A 38-year-old man presents to the primary care clinic for a 6-week follow-up. He was recently diagnosed with Helicobacter pylori-associated peptic ulcer disease (PUD) after an upper endoscopy. At his last visit, he was placed on omeprazole 40 mg BID and bismuth subcitrate potassium 140 mg/metronidazole 145 mg/tetracycline 125 mg four times daily for 14 days. Today, he states his dyspepsia has significantly improved, and he is no longer having any other gastrointestinal symptoms. Vital signs are

Correct answer: B) Perform urea breath test for H. Pylori Explanation: Peptic ulcer disease (PUD), a break in the lining of the stomach or intestinal lining, has two major causes: Helicobacter pylori (H. pylori) infection or long-term NSAID use. Symptoms of PUD include gnawing or aching epigastric pain, dyspepsia, and nausea. Nighttime pain is not uncommon with this disease. While food consumption usually causes an increase in pain in patients with gastric ulcers, those with duodenal ulcers often have a decrease in their pain with eating. PUD is usually diagnosed by upper endoscopy, and treatment is aimed at eradicating the underlying causal agent. All patients with PUD should be initiated on antisecretory therapy along with a proton pump inhibitor (PPI). The length of treatment depends on the etiology, risk factors, and past medical history of the patient. In the case of an active H. pylori infection, as in the patient above, a standard bismuth quadruple therapy (bismuth subsalicyla

A 55-year-old man with a medical history of hypertension presents to the emergency department with 2 days of left lower quadrant pain and a fever of 38.2°C at home. He denies nausea or vomiting and has been able to tolerate oral intake at home. He reports some intermittent painless hematochezia over the last year but states that he has never had a colonoscopy. His vitals are HR 89 beats per minute, RR 16 breaths/min, and BP 142/90 mm Hg. His abdominal exam shows moderate tenderness to palpation

Correct answer: B) discharge home with follow-up in 48 hr Explanation: Diverticulosis is one of the most common causes of painless hematochezia in older adults. Approximately 4% of individuals with diverticulosis will go on to develop diverticulitis, an inflammation of the colonic wall that classically presents with lower left quadrant abdominal pain. The majority of diverticulitis involves the sigmoid colon, mimicking the symptoms of appendicitis but on the left side of the abdomen. Patients with uncomplicated acute diverticulitis may be discharged home without oral antibiotics with close follow-up in the next 48 hours. If antibiotics are indicated, ciprofloxacin and metronidazole or trimethoprim-sulfamethoxazole and metronidazole are two commonly used regimens. These patients should be given clear return precautions for worsening fever or abdominal pain as this could indicate treatment failure or the development of complications

Which of the following is considered the gold standard in the treatment of chronic anal fissures? A) Cryosurgery B) Lateral internal sphincterotomy C) Rubber band ligation D) Sclerotherapy

Correct answer: B) later internal sphincterotomy Explanation: Lateral internal sphincterotomy has been widely accepted as the treatment of choice and gold standard for chronic anal fissures, resulting in healing of > 90% of patients. However, anal sphincterotomy is usually reserved for refractory anal fissures because it carries an increased risk of incontinence. The goal of surgical therapy is to relax the internal anal sphincter. The procedure involves division of the internal anal sphincter from its distal-most end, up to the dentate line. Acute anal fissures often heal spontaneously; treatment should start with warm sitz baths for symptom relief, increased dietary fiber to soften stools, and topical anesthetics to decrease pain with bowel movements. If an acute anal fissure fails conservative therapy or becomes chronic, then surgical or medical procedures can be initiated. The high rates of incontinence reported after surgical sphincterotomy have led to the study and implementation of alternative medical treatments, mainly botulinum toxin, organic nitrate preparations and topical calcium channel blockers which inhibit spasm or increase blood flow to the fissure. Question: What physical exam findings are pathognomonic for a chronic anal fissure? Answer: External skin tags, or sentinel pile, and hypertrophied anal papillae.

Which of the following patients meet criteria for metabolic syndrome? A) A 35-year-old man with a waist circumference of 108 cm, triglycerides of 130 mg/dL, and blood pressure of 140/90 mm Hg B) A 40-year-old woman with a body mass index of 35, triglycerides 150 mg/dL, and fasting glucose of 110 mg/dL C) A 45-year-old man with a waist circumference of 110 cm, triglycerides of 155 mg/dL, and fasting glucose of 120 mg/dL D) A 50-year-old man with waist circumference of 110 cm, triglycerides of 1

Correct answer: C) A 45-year-old man with a waist circumference of 110cm, triglycerides of 155 mg/dL, and fasting glucose of 120 mg/dL Explanation: Metabolic syndrome refers to a group of findings that increase the chance of developing both type 2 diabetes and atherosclerotic cardiovascular disease, which in turn increases an individual's cardiovascular risk. According to the National Cholesterol Education Program, metabolic syndrome is defined by any three of the following criteria: (1) Waist circumference of > 102 cm for men and > 88 cm for women, (2) Triglycerides of > 150 mg/dL, high density lipoprotein (HDL) cholesterol of < 40 for men and < 50 mg/dL for women, blood pressure of > 130/ > 85 mm Hg, and (3) Fasting glucose of > 100 mg/dL. Therefore, the 45-year-old man with a waist circumference of 110 cm, triglycerides of 155 mg/dL, and fasting glucose of 120 mg/dL meets criteria for metabolic syndrome. Treatment of metabolic syndrome begins with lifestyle modifications such a

A 53-year-old man presents with abdominal pain for 2 days that he localizes to the left side of the abdomen. He has associated nausea and loss of appetite. He reports that his last bowel movement was yesterday and was normal for him. His medical history is significant for obesity and type 2 diabetes mellitus. A physical examination is performed, and there is left lower quadrant tenderness to palpation without peritoneal signs. Bowel sounds are auscultated and consist of clicks and gurgles at a r

Correct answer: C) Admit for IV ABX, IV fluids, and bowel rest Explanation: Diverticula are outpouches of the circular muscle of the colon wall where the vasculature penetrates the muscle. Risk factors for developing diverticulosis include low dietary fiber, high dietary intake of red meat, tobacco use, obesity, and lack of physical activity. Diverticulitis describes inflammatory changes of the herniated areas. Clinical manifestations are dependent on the severity of the inflammatory process. Abdominal pain, nausea, vomiting, and low-grade fever are common symptoms. Typically, the abdominal pain is in the left lower quadrant of the abdomen since diverticulosis primarily involves the descending and sigmoid colon. Complications of diverticulitis include abscess formation, partial colonic obstruction, fistula formation, or perforation of the bowel. Perforation allows fecal contamination of the peritoneum. CT scan of the abdomen should be done to examine for complications of diverticulit

Which of the following organisms is the most common cause of traveler's diarrhea? A) Campylobacter jejuni B) Enterohemorrhagic Escherichia coli C) Enterotoxigenic Escherichia coli D) Giardia lamblia

Correct answer: C) Enterotoxigenic Escherichia Coli Explanation: While viruses are responsible for most infectious diarrheas in the United States, travel to a resource-limited country is associated with an 80% probability of bacterial diarrhea. The most common cause of traveler's diarrhea is enterotoxigenic Escherichia coli, which is acquired by ingesting fecally contaminated food and water. Countries in Asia, Africa, Latin America, and parts of the Middle East are high-risk destinations. Other risk factors include travel during the rainy season and type of travel (e.g. camping, living with native inhabitants). Patients present with abrupt onset of crampy abdominal pain and watery diarrhea. Treatment consists of oral rehydration, a 3-day course of ciprofloxacin, and antimotility agents, such as loperamide, may be used under certain circumstances.

A 27-year-old woman presents to her primary care physician with dysphagia. Her symptoms began several months ago. She has trouble swallowing solids and liquids, though liquids seem to make her choke and sputter the most. Therefore, she has been unable to eat and has thus experienced significant weight loss. She has no significant past medical history apart from a 20 pack-year smoking history. She reports no recent travel. Physical examination is within normal limits. A barium esophagram reveals

Correct answer: C) Esophageal manometry Explanation: Achalasia is a motor disorder of the distal esophagus resulting from degeneration of the Auerbach plexus. It is the most common motility disorder and is often found in patients younger than 50 years of age. The lower esophageal sphincter fails to relax during swallowing. As a consequence, natural peristalsis is disrupted and the patient experiences dysphagia to solids and liquids, with liquids often most problematic. A barium esophagram is helpful in making the diagnosis and should reveal the classic bird beak, tapering at the esophageal sphincter. If a barium swallow is suggestive of achalasia, esophageal manometry should be performed to confirm the diagnosis. Esophageal manometry will reveal elevated resting lower esophageal sphincter pressure, incomplete lower esophageal sphincter relaxation after swallowing, and almost total absence of peristalsis in the esophageal body.

Which of the following is the most sensitive diagnostic study for diagnosing achalasia? A) Barium swallow B) Electromyography C) Esophageal manometry D) Esophagogastroduodenoscopy

Correct answer: C) Esophageal manometry Explanation: Esophageal manometry is the most sensitive diagnostic study for the diagnosis of achalasia. Achalasia is a primary esophageal motility disorder characterized by loss of peristalsis in the distal esophagus and a failure of the lower esophageal sphincter to relax with swallowing. Achalasia is most commonly diagnosed in patients 25-60 years of age. Achalasia affects men and women equally. Achalasia is caused by degeneration of the neurons in the esophageal wall, resulting in loss of inhibitory innervation in the lower esophageal sphincter. This loss causes the basal sphincter pressure to rise and renders the sphincter incapable of normal relaxation. Patients with achalasia typically present with insidious onset and progressive dysphagia for solids and liquids. Regurgitation of undigested food and heartburn are other common symptoms of achalasia. Most patients with achalasia have difficulty belching. The diagnosis of achalasia is based on esophageal manometry demonstrating aperistalsis in the distal two-thirds of the esophagus and incomplete lower esophageal sphincter relaxation. Other diagnostic studies may include a chest radiograph, upper endoscopy, and barium swallow, which may reveal the pathognomonic "bird-beak" appearance of the lower esophageal sphincter. Treatment options for achalasia include pneumatic dilation, surgical myotomy, botulinum toxin injection, oral nitrates, or calcium channel blockers. Pneumatic balloon dilation and myotomy have the highest success rates but often require repeat treatment within 10 years. Botulinum toxin injection into the lower esophageal sphincter is usually reserved for high-risk surgical patients.

A 52-year-old man comes to the clinic complaining of difficulty swallowing, upper abdominal pain, and heartburn. He says he has a difficult time swallowing solid foods and the food frequently gets "stuck" in his throat. He has a history of food allergies, asthma, and atopic dermatitis. Laboratory studies show markedly elevated serum IgE levels. Upper endoscopy shows stacked circular rings. Which of the following is the most likely diagnosis? A) Achalasia B) Esophageal stricture C) Esopha

Correct answer: C) Esophagitis Explanation: Dysphagia to solids, upper abdominal pain, GERD-like symptoms (eg, heartburn), food impaction, and a history of allergies suggests eosinophilic esophagitis (EE). EE is an allergic inflammatory condition of the esophagus that primarily involves eosinophils. EE is strongly associated with allergic diseases (eg, food allergies, asthma, atopic dermatitis). Laboratory studies typically show elevated levels of serum IgE. Endoscopic findings typically show stacked circular rings that can be transient or fixed along with white nodules with granularity. Treatment usually consists of dietary modification (avoidance of food allergens), medical therapy (treatment with corticosteroids and other anti-inflammatory agents), and mechanical dilation of the esophagus.

Perforation of what gastrointestinal structure is associated with the highest mortality? A) Cecum B) Duodenum C) Esophagus D) Stomach

Correct answer: C) Esophagus Explanation: Perforation of any viscus is associated with significant morbidity and mortality. Perforation of the esophagus is most often diagnosed late, resulting in a fulminant course ending in death. Anatomically, the esophagus lacks a serosal layer, leading to immediate contamination of the mediastinum with most esophageal perforations. Perforations can occur from various causes, including forceful vomiting, childbirth, cough, heavy lifting, endoscopy, and nasogastric tube placement. More than 90% of spontaneous esophageal ruptures occur in the distal esophagus. In contrast, rupture resulting from blunt trauma to the neck or thorax usually occurs in the proximal and middle thirds of the esophagus. Treatment includes early initiation of broad-spectrum antibiotics, such as piperacillin-tazobactam. Patients should be kept NPO, and early surgical consultation is warranted.

A 52-year-old woman presents to her primary care clinic with fatigue and pruritus for the past 2 months. Vital signs today include a HR of 82 bpm, BP of 130/80 mm Hg, RR of 20/min, oxygen saturation of 99% on room air, and T of 98.6°F. Physical examination reveals scleral icterus and scattered excoriations. Her abdomen is soft and nontender. Laboratory findings include a white blood cell count of 7,000/μL, lipase of 85 U/L, aspartate aminotransferase of 52 U/L, alanine aminotransferase of 65 U/L, alkaline phosphatase of 290 U/L, total bilirubin of 3.5 mg/dL, and antimitochondrial antibody titers of 1:100. Which of the following is the recommended initial pharmacologic treatment of the suspected diagnosis? A) Bezafibrate B) Glecaprevir-pibrentasvir C) Obeticholic acid D) Prednisone E) Ursodeoxycholic acid

Correct answer: E) Ursodeoxycholic acid Explanation: Primary biliary cholangitis is a chronic progressive autoimmune disease that affects the intrahepatic bile ducts. It causes chronic cholestasis (decreased flow of bile) and can result in cirrhosis. It is much more common in women (approximately 90%) and is most often diagnosed between 30 and 65 years of age. The diagnosis of primary biliary cholangitis may be initially suspected due to abnormal liver biochemical tests in asymptomatic patients or patients may have clinical manifestations from cholestasis, such as pruritus, fatigue, right upper quadrant discomfort, and jaundice. Exam findings can include excoriations, jaundice, and hepatosplenomegaly. It is common for patients with primary biliary cholangitis to have other autoimmune diseases, such as Sjögren syndrome. Primary biliary cholangitis should be suspected in adults who have an elevated alkaline phosphatase without extrahepatic biliary obstruction. Other supportive laboratory findings include elevated antimitochondrial antibody, elevated antinuclear antibody, elevated high-density lipoprotein, elevated gamma-glutamyl transpeptidase, and elevated 5-nucleotidase. The diagnosis of primary biliary cholangitis is established in patients who have at least two of the following in the absence of extrahepatic biliary dilation or comorbid liver disease: an alkaline phosphatase elevated to at least 1.5 times the upper limit of normal, antimitochondrial antibody titers elevated to at least 1:40, and histologic evidence of primary biliary cholangitis (nonsuppurative destructive cholangitis and destruction of interlobular bile ducts). Patients can be evaluated for extrahepatic biliary dilation with a right upper quadrant abdominal ultrasound. Liver biopsy is often unnecessary to make the diagnosis. All patients diagnosed with primary biliary cholangitis should be referred to a hepatologist. It is important for patients who are diagnosed with primary biliary cholangitis to be vaccinated against hepatitis A virus and hepatitis B virus. They should also have the pneumococcal vaccine and other routinely recommended vaccines, such as the annual influenza vaccine. Patients with primary biliary cholangitis should abs

A healthy 6-year-old boy presents to the ED with bloody diarrhea. He was in his usual state of health until one week ago when loose, watery stools (up to 10 per day) were noted. He was seen by his pediatrician four days ago but has since developed increasing amounts of blood and pus in his stools along with a low-grade fever. Mom states there is no recent travel, antibiotic use, or known sick contacts. His vitals are a heart rate 118 beats per minute, oxygen saturation 100% on room air, and rect

Correct answer: C) Extraintestinal manifestations such as hallucinations, confusion, and seizures may occur. Explanation: Shigella species cause an invasive diarrhea that rarely infects infants younger than 3-months-old and is most common between 2 and 3 years of age. Infection is typically transmitted by person-to-person (fecal-oral) contact or through ingestion of contaminated material. Clinical illness varies from mild to severe, with some patients exhibiting abdominal cramps and tenderness. Dysentery (diarrhea with significant blood, pus, and mucus) occurs in approximately 33% of patients. Some patients may also develop extraintestinal manifestations such as reactive arthritis, seizures, and hallucinations. The general management of diarrhea from a bacterial source is oral rehydration and avoidance of anti-diarrheal agents. Antibiotics should not be given to young, healthy individuals and should be reserved for immunocompromised, bacteremic, or hospitalized patients.

A 31-year-old nurse presents to the emergency department after being stuck with a blood-filled hollow bore needle during a resuscitation. She discovers the patient has hepatitis B. That same day, she has her blood drawn. Which of the following serologic markers indicates prior hepatitis B immunization? A) Hepatitis B core antibody (anti-HBc) B) Hepatitis B envelope antigen (HBeAg) C) Hepatitis B surface antibody (anti-HBs) D) Hepatitis B surface antigen (HBsAg)

Correct answer: C) Hepatitis B surface antibody (anti-HBs) Explanation: Presence of the hepatitis B surface antibody (anti-HBs) immediately postexposure indicates prior immunization. Although anti-HBs may also be seen with acute infection, its appearance is associated with a postexposure lag time of roughly two months. A person negative for HBsAg but positive for anti-HBs either has cleared an infection or has been vaccinated previously. The vaccination for hepatitis B contains only the surface antigen, thus an immunized patient will only have surface IgG present if they have been vaccinated. -The hepatitis B core antibody (anti-HBc) (A) is the earliest antibody to develop in response to acute hepatitis B virus (HBV) infection, appearing predominantly as IgM anti-HBc at about six to eight weeks after infection. The anti-HBc typically persists for life, but after about six months, the total anti-HBc mainly consists of IgG rather than IgM. Shortly after the appearance of the HBsAg in an acute infection, the hepatitis B e antigen (HBeAg) (B) appears. In general, the presence of HBeAg is associated with rates of viral replication and increased infectivity, however, some variants of hepatitis B virus do not produce the e antigen. During the natural course of an infection, the HBeAg may be cleared, and antibodies to the e antigen (anti-HBe) will arise immediately afterward. This conversion is usually associated with a dramatic decline in viral replication and infectivity. The hepatitis B surface antigen (HBsAg) (D) is used to screen for the presence of acute hepatitis B infection. It is the first detectable viral antigen to appear during infection. However, early in an infection, this antigen may not be present; it may also be undetectable later if the infection has been adequately cleared by the host. During this window in which the host remains infected but is successfully clearing the virus, IgM antibodies to the hepatitis B core antigen (anti-HBc IgM) may be the only serological evidence of disease.

You examine a 17-year-old girl due to anorexia nervosa. She has low self-esteem with intense fear of gaining weight. She restricted her diet for the past three months. On physical exam, her BMI is 18 with heart rate of 35 beats per minute at rest. You decide to admit her for inpatient management. You plan to start nutritional therapy but worry about the risk of refeeding syndrome. Which laboratory finding is most consistent with refeeding syndrome? A) Hyperkalemia B) Hypermagnesemia C) Hypoph

Correct answer: C) Hypophosphatemia Explanation: The core features of anorexia nervosa are restriction of energy intake, intense fear of gaining weight, and distorted perception of body weight. Patients with food-restricting subtype of anorexia nervosa should be evaluated for complications. Refeeding syndrome is a complication that occurs during nutritional therapy of malnourished patients. It is marked by hypophosphatemia, hypokalemia, thiamine deficiency, congestive heart failure and peripheral edema. Hypophosphatemia originates when phosphate stores are depleted during starvation. When nutritional replenishment begins and patients are fed with carbohydrates, glucose causes release of insulin that triggers cellular uptake of phosphate. Insulin also causes cells to produce adenosine triphosphate (ATP) and 2, 3-diphosphoglycerate (2, 3 DPG) that require phosphate, therefore further depleting phosphate stores. Complications of the syndrome may be reduced by slowing the rate of nutriti

An 80-year-old man who presents to the emergency department with a complaint of sudden onset left-sided abdominal pain and bloody diarrhea. Abdominal CT demonstrates thickening of the bowel wall and free peritoneal fluid. What is the most likely diagnosis? A) Diverticulitis B) Infectious colitis C) Ischemic colitis D) Small bowel obstruction

Correct answer: C) Ischemic colitis Explanation: Ischemic colitis is the sudden reduction of blood flow to the colon usually caused by an arterial occlusion, venous thrombosis, or hypoperfusion. Ischemic colitis is the most common form of ischemic bowel disease and most often affects the elderly. Blood to the colon is supplied by the mesenteric vasculature and when this is compromised, a reduction in blood flow occurs causing mesenteric and colonic ischemia. Patients with ischemic colitis present with sudden onset of mild to severe cramping, often on the left side of the abdomen, along with rectal bleeding or bloody diarrhea within 24 hours of symptom onset. An abdominal CT may demonstrate findings of thickening of bowel wall or free peritoneal fluid and colonoscopy is diagnostic. Treatment depends on the severity and complexity of the disease, ranging from supportive care to antithrombotic therapy to surgical exploration. Complications include necrosis of the bowel and gangrene, whi

A 66-year-old woman has been on estrogen replacement therapy for 9 years. Her social history is significant for 20 years of heavy alcohol use and 30 years of tobacco use. Her family history is positive for familial adenomatous polyposis. Which of the following organs is most likely to undergo malignant transformation due to this patient's alcohol use? A) Breast B) Colorectal C) Liver D) Lung

Correct answer: C) Liver Explanation: Hepatocellular carcinoma, also known as malignant hepatoma, is a primary cancer of the liver. It is rare in the United States, where most liver cancer is secondary from a metastatic site. However, in countries where hepatitis is endemic, such as China, this cancer represents the most common of all liver cancers. The most common causes are viral hepatitis (hepatitis B and C) and alcohol-related cirrhosis. Other risk factors include aflatoxin exposure, hemochromatosis, Wilson disease, and type 2 diabetes mellitus. Approximately 20-25% of cases are due to hepatitis B or C infection, while 80% of cases are due to alcohol-related cirrhosis. Alpha-fetoprotein is elevated in the majority of cases, but it is nonspecific. Triple or quad-phase liver CT scan aids in the diagnosis. Biopsy provides a definitive diagnosis. Ultrasound plays an important role in the surveillance of high-risk patients. However, contrast-enhanced abdominal CT with triple-phase scanning remains the primary diagnostic tool. Treatment options include surgical resection, transplantation, chemotherapy, and radiotherapy.

A 41-year-old woman presents to the emergency department with an 8-hour history of left-sided epigastric pain that radiates to her back. She vomited four times overnight after going out to eat. The patient's vitals are BP of 105/70 mm Hg, HR of 112 bpm, T of 99.4°F, RR of 20/min, and oxygen saturation of 98% on room air. Urine is negative for pregnancy but positive for glucose. She has epigastric tenderness but not rigidity and cannot lie down due to pain. You order a 2 L bolus of saline with an opioid for pain. Laboratory results show a serum glucose of 280 mg/dL, serum amylase of 240 U/L, and serum lipase of 311 U/L. Liver enzymes are within normal limits. Bedside ultrasound of the gallbladder is negative for cholelithiasis. Patient is admitted to the surgery floor, and pain has improved significantly after 2 days. Additional blood work shows a triglyceride level of 1,090 mg/dL. What key management should be discussed with her besides diabetic medication and dietary changes? A) Elective cholecystectomy B) Extended-release niacin therapy C) Long-term fenofibrate D) Long-term rosuvastatin E) Trial of disulfiram

Correct answer: C) Long term fenofibrate Explanation: The patient shows signs and symptoms of acute pancreatitis due to hypertriglyceridemia. Central or left-central chest pain that radiates to the back is a classic sign of pancreatitis. Patients with pancreatitis also prefer to sit up or lean forward. Nausea and vomiting are common in pancreatitis from all etiologies. Fever, tachycardia, and hypotension are common in more severe pancreatitis. Such signs may add up to systemic inflammatory response syndrome and would require hospitalization. Jaundice is sometimes present. Serum amylase (reference range: 50-140 U/L) and lipase (0-160 U/L) are the two key labs that show elevation in pancreatitis. Treatment of acute pancreatitis involves aggressive fluid replacement, pain relief with an intravenous opioid, and NPO status until stabilization. Repeat complete metabolic panels are necessary with special attention to glucose, electrolytes, and calcium levels, as some patients are hypocalcemic. The main etiologies of acute pancreatitis include gallstones that cause obstruction of the ductal system (including the ampulla) and reflux of bile and pancreatic enzymes or edema of these structures from the passing of stones, also causing obstruction, stasis, and reflux; chronic alcohol use; and hypertriglyceridemia, possibly causing the capillaries of the pancreas to become blocked. Gallstones and alcohol use are less likely in this case, given the normal liver enzymes and ultrasound findings. By contrast, elevated levels of serum triglyceride is a direct clue to her hypertriglyceridemia. Triglyceride levels > 500 mg/dL pose a risk for acute pancreatitis, with the risk increasing for those with levels > 1,000 mg/dL or prior episodes. The most effective agents for treatment of hypertriglyceridemia are fibrates, such as fenofibrate or gemfibrozil (the former is preferred). Acute pancreatitis caused by hypertriglyceridemia is most often associated with uncontrolled diabetes. Controlling diabetes, providing dietary counseling, and investigating potential drug-related causes (e.g., estrogen) are also important. More severe pancreatitis cases have signs of infection (fever, elevated white blood cell count) and pancreatic ne

A patient of yours with a history of diverticulosis develops acute diverticulitis. He is admitted to the hospital for inpatient care. He is successfully treated and discharged home. He follows up with you a few days later. Which of the following should you most likely recommend to him at this time? A) Colonoscopy within 3 days B) High fiber diet until 6 weeks of no symptoms C) Low fiber diet until 6 weeks of no symptoms D) Sigmoidoscopy within 3 days

Correct answer: C) Low fiber diet until 6 weeks of no symptoms. Explanation: Diverticulosis refers to herniations through the mucosa and submucosa of the colonic wall. It is associated with a low fiber diet. It occurs mainly in the sigmoid colon. Patients are usually asymptomatic, but 10-25% will develop diverticulitis and about 10% will develop hemorrhage. Diverticulitis stems from retention of food and bacteria in the herniations, which leads to fecaliths, obstruction, compromise of the mucosal blood supply, infection, and perforation. This causes left lower quadrant pain, fever, nausea, constipation, and vomiting. Abdominal computed tomography is the imaging modality of choice. Treatment may include outpatient antibiotics, but in more complicated cases (e.g. evidence of abscess or perforation), inpatient care is necessary with intravenous fluids, antibiotics, and possibly surgery. A low fiber diet is recommended after an episode of acute diverticulitis until the patient remains asymptomatic for 6 weeks. A low fiber diet is especially effective in cases of acute diverticulitis, because it helps reduce the frequency of stools and allows the affected portion of the colon to adequately heal.

Which of the following findings is classically associated with pyloric stenosis? A) Bilious vomiting B) Elevated lipase C) Metabolic alkalosis D) Poor feeding

Correct answer: C) Metabolic alkalosis Explanation: Vomiting from pyloric stenosis classically causes a hypochloremic metabolic alkalosis. Hypertrophic pyloric stenosis is the most common cause of obstruction in the first month of life. It is more common in boys and typically presents with projectile vomiting that occurs promptly after feeds. Patients typically present 2 to 6 weeks after birth. If vomiting is prolonged, it can lead to hypochloremic metabolic alkalosis secondary to loss of fluids from the stomach. Physical examination may reveal an olive-shaped mass representing the hypertrophied pyloric sphincter. Diagnosis is made with either abdominal ultrasound or upper GI series.

You are evaluating a 73-year-old woman with a history of weakness, fatigue, and difficulty walking. She lives alone and unassisted. She has no past medical history and takes no medications. Physical examination shows an enlarged, smooth, and tender tongue. Pallor of the conjunctivae is also seen. Which of the following laboratory studies would be most likely to confirm the diagnosis? A) Iron panel B) Mean corpuscular volume C) Methylmalonic acid and homocysteine levels D) Peripheral blood sme

Correct answer: C) Methylmalonic acid and homocysteine levels Explanation: This patient most likely has vitamin B12 (cobalamin) deficiency. Vitamin B12 deficiency typically presents with paresthesias (eg, numbness and tingling), ataxia, glossitis, and megaloblastic anemia (eg, weakness, fatigue, easy bruising). Laboratory studies will show increased serum (and urine) concentrations of both methylmalonic acid (MMA) and homocysteine (HC). These increased levels occur as a result of their decreased rate of metabolism. In contrast, isolated levels of HC is only seen in folate deficiency. Thus, this test is critical in differentiating the specific vitamin deficiency as the etiology of the anemia.

You are called to examine a 3-year-old boy in the emergency department for possible ingestion. He was found by his father drooling and playing with an opened drain cleaner. After suspecting a caustic ingestion, he immediately called poison control and was advised to bring the boy to the emergency department. The boy is irritable and drooling. His physical examination is otherwise normal. Which of the following is the most appropriate next step in management? A) Give activated charcoal B) Give

Correct answer: C) Order an upper endoscopy Explanation: Caustic materials cause tissue injury. Caustic alkaline materials are found in drain cleaners, various cleaning agents, hair relaxers, dishwasher agents, and disk batteries. Alkalis produce liquefaction necrosis that allows further tissue penetration of the toxin and setting the stage for possible perforation. On the other hand, acids in household products include toilet bowl cleaners, swimming pool cleaners, and rust removers. Acids produce a coagulative necrosis that limits further tissue penetration, though perforation can still occur. Ingestion of caustic materials can produce injury to the oral mucosa, esophagus, and stomach. The symptoms include pain, drooling, vomiting, abdominal pain, and difficulty swallowing. Initial treatment of caustic exposures includes thorough removal of the product from the skin or eye by flushing with water. Endoscopy should then be performed within 12 to 24 hours of ingestion in symptomatic pat

Which of the following groups of patients should be screened for hepatitis C? A) Children starting daycare B) College students living in a dorm C) Nursing home residents D) People aged 18 to 79 years

Correct answer: D) People aged 18-79 years Explanation: People aged 18 to 79 years should be screened for hepatitis C. Hepatitis C is a treatable illness if detected early. If left untreated it can lead to liver cirrhosis and end-stage liver disease. Patients who should be screened for Hepatitis C infection include those who were born between 1945 and 1965, HIV patients, dialysis patients, incarcerated individuals, patients with a history of illicit injection drug use or intranasal cocaine use, pregnant patients, and patients who received potentially contaminated blood products or tissue. Patients who have a known exposure, such as a needle stick or a partner with Hepatitis C should also be screened. The initial screening test for chronic HCV infection in children and adults is an HCV antibody test. A reactive antibody test should be followed by an HCV RNA test. A nonreactive antibody test generally indicates the absence of HCV infection.

A 7-year-old girl has a history of cerebral palsy, static encephalopathy, seizure disorder, and recent aspiration pneumonia. She completed treatment for pneumonia two days ago but now has developed foul-smelling, watery diarrhea and abdominal cramping. Her vital signs are within normal limits for age. What is the treatment of choice? A) Intravenous clindamycin B) Intravenous vancomycin C) Oral metronidazole D) Reassurance regarding antibiotic-associated diarrhea

Correct answer: C) PO Metronidazole Explanation: The onset of diarrhea and vomiting during or closely following a course of antibiotics raises concern for Clostridium difficile infection. Aspiration pneumonia is commonly treated with clindamycin or amoxicillin-clavulanate, both of which may alter the intestinal microbiome and predispose to overgrowth of toxigenic C.difficile. However, any antibiotic may predispose children to C.difficile infection. C.difficile colitis is caused by an intestinal overgrowth of toxigenic C.difficile, followed by production of C.difficile toxin and invasion of the intestinal epithelium. It is best diagnosed by stool toxin PCR. The treatment of choice for mild-to-moderate C.difficile infection in children is oral vancomycin, an alternative is oral metronidazole, however oral vancomycin is more effective. Patients may also require intravenous hydration if oral hydration is not sufficient to replace stool output, which may be copious.

A 32-year-old man presents with right upper quadrant pain that started earlier today. His symptoms initially included mild generalized abdominal discomfort and nausea before localizing to the right side of his abdomen. He has a low-grade fever but his vital signs are otherwise unremarkable. He has minimal tenderness in the right lower quadrant to palpation. Which of the following signs would be indicative of a retrocecal location of the appendix? A) Murphy sign B) Obturator sign C) Psoas sign

Correct answer: C) Psoas sign Explanation: Appendicitis results from acute obstruction of the appendiceal lumen. This is most commonly caused by an appendicolith, but may also be due to calculi, tumor, enlarged lymph nodes, or parasites. Appendicitis is common, with 7% of the population developing the disease at some point in their lives. It is most commonly seen in adolescents and young adults. The classic presentation of acute appendicitis is a patient who develops vague periumbilical pain, anorexia, nausea, vomiting, and low-grade fever, followed by migration of pain to the right lower quadrant. However, the presentation is highly variable. McBurney point, an area 2 cm from the anterior superior iliac spine on a line connecting the umbilicus to this landmark, is the classic area of tenderness in appendicitis. However there is significant anatomic variation in the location of the appendix, and patients may have a pelvic or retrocecal appendix, and do not have tenderness at McBurney

An 85-year-old nursing home patient presents with diffuse abdominal pain and distension with nausea but no vomiting. The above abdominal radiograph is obtained. What is the most appropriate management? A) Enema B) Nasogastric tube and bowel rest C) Sigmoidoscopy D) Surgical intervention

Correct answer: C) Sigmoidoscopy Explanation: The radiograph demonstrates a markedly dilated single loop of colon consistent with sigmoid volvulus. This is a closed-loop obstruction that results from the twisting of a mobile segment of bowel. These occur almost entirely in two populations: older, bedridden patients with debilitating comorbid disease and patients of any age with profound neurologic or psychiatric illness. Almost all patients have a history of chronic severe constipation. Although spontaneous reduction of a sigmoid volvulus can occur, it is infrequent enough to mandate procedural intervention. Sigmoidoscopy is used to decompress and detorse the bowel.

Which of the following statements is most correct regarding total parenteral nutrition in a patient with Crohn disease? A) Patients tolerate enteral feedings well after total parenteral nutrition is discontinued B) Total parenteral nutrition carries the risk of deep vein thrombosis and cardiomyopathy C) Total parenteral nutrition is an alternative to chronic low dose steroids D) Total parenteral nutrition is not recommended in those with short bowel syndrome

Correct answer: C) TPN is an alternative to chronic low dose steroids Explanation: Crohn's disease is a chronic, inflammatory disease of the bowel involving any part of the gastrointestinal tract (mouth to anus), but with a predilection for the distal ileum and colon. The mucosal inflammation of Crohn's disease is transmural, leading to the formation of fistulas, abscesses, and stricture formation. Affected segments of bowel are often separated by segments of non-diseased bowel and are sometimes referred to as "skip lesions". Symptoms of Crohn's disease can include chronic diarrhea, fever, weight loss, and abdominal pain. Diagnosis is with colonoscopy, upper endoscopy, CT enterography or magnetic resonance enterography. Crohn's disease can be divided into four disease states, with each state necessitating differing treatment strategies. The most mild state is asymptomatic remission, where patients have no symptoms either after successful medical or surgical management, or spontan

A 56-year-old man presents to the clinic reporting achy abdominal pain for the past 3 days along with mild nausea and loose stools. He reports that the abdominal pain can get very severe. He does not remember eating anything new or different recently. His temperature is 100.8°F, heart rate is 102 bpm, blood pressure is 132/68 mm Hg, respiratory rate is 22/minute, and oxygen saturation is 98%. A physical exam reveals some left lower quadrant tenderness with a palpable mass. There is no evidence

Correct answer: CT of the abdomen/pelvis Explanation: A complete blood count may reveal mild to moderate leukocytosis. Patients with severe disease will present with severe, generalized abdominal pain with peritoneal signs (e.g., abdominal distension and guarding). Patients with mild disease are treated empirically without requiring additional imaging. After the acute phase resolves and the patient is no longer symptomatic, a colonoscopy, computed tomography (CT) of the colon, or barium enema should be ordered to confirm the empiric diagnosis and rule out neoplasms. For patients who do not respond to empiric therapy after several days or those with severe disease (in this case, leukocytosis), a CT scan of the abdomen (anterior/posterior view) is the imaging tool of choice to diagnose the condition, determine the severity of the disease, and rule out complications or other differential diagnoses. Findings on a CT scan that implicate diverticulitis include the presence of colonic divert

A 30-year-old woman with no prior medical history presents to the emergency department with abdominal pain, nausea, and vomiting for 2 days. Vital signs today include a HR of 111 bpm, BP of 126/70 mm Hg, RR of 20/min, oxygen saturation of 97% on room air, and T of 99.1°F. Physical examination reveals the patient to be actively vomiting with diffuse abdominal tenderness and upper abdominal distention. Laboratory findings include a white blood cell count of 10,000/μL, lipase of 250 U/L, aspartat

Correct answer: Cecal volvulus Explanation: Cecal volvulus is a type of large bowel obstruction marked by rotation or torsion of a mobile cecum and ascending colon. In most cases of cecal volvulus, there is an axial rotation of the cecum and a portion of the terminal ileum, which causes twisting of the mesentery and blood vessels supplying the cecum. However, in some cases, the cecum and a portion of the terminal ileum fold upward without an axial twisting. Cecal volvulus manifests clinically with symptoms and signs of bowel obstruction, including abdominal pain, nausea, vomiting, and obstipation. The abdominal pain is often described as constant with intermittent cramping. Exam findings include diffuse abdominal tenderness and abdominal distention. Patients with cecal volvulus complicated by bowel perforation may have fever, hypotension, and peritoneal signs (guarding and rebound tenderness). The recommended initial imaging study is an upright abdominal plain film to evaluate for ob

To properly treat an individual with phenylketonuria, a life-long strict adherence to a diet consisting of which of the following is recommended? A) High in beans, low in vegetables B) High in meats, low in beans C) High in starches, low in fruits D) High in vegetables, low in meats

Correct answer: D) High in vegetables, low in meats Explanation: Phenylketonuria (PKU) is an autosomal recessive inborn error of metabolism characterized by a mutation in the gene that codes for phenyalanine hydroxylase. This liver enzyme normally converts the amino acid phenyalanine into tyrosine. A deficiency of the enzyme results in high levels of phenylalanine and low levels of tyrosine. The resultant accumulation of phenylalanine is converted to phenylketones which can be detected in the urine. Most countries include this test in a newborn screen which is obtained a few days after birth. If not detected, a neonate with this disease will present with seizures, albinism and musty sweat and urine (due to the presence of ketones). Left untreated, the neonate will develop microcephaly, hyperactivity, developmental delay or failure, mood disorders, motor control abnormalities and learning disabilities. If not screened, diagnosis is confirmed when elevated serum phenyalanine or urine p

A 3-week-old infant presents with projectile vomiting. His parent reports he has vomited after each feed for the last 24 hours. What electrolyte abnormality do you expect to see? A) Hyperchloremic, hyperkalemic, metabolic alkalosis B) Hyperchloremic, hypokalemic, metabolic alkalosis C) Hypochloremic, hyperkalemic, metabolic alkalosis D) Hypochloremic, hypokalemic, metabolic alkalosis

Correct answer: D) Hypochloremic, hypokalemic, metabolic alkalosis Explanation: Hypertrophic pyloric stenosis is one of the most common causes of intestinal obstruction in infants beyond 1 month of age. Classically, infants present between 2 and 6 weeks of age with progressive emesis that ultimately becomes projectile. Since the stenosis is at the level of the pylorus, emesis is nonbilious. Infants finish feeding and then regurgitate the entire volume and continue to be hungry. On examination of the abdomen, the hypertrophic pylorus may be palpable in the upper abdomen and is often described as an olive. With continued vomiting, infants lose both hydrogen and chloride ions present in the gastric juices. As the condition continues, kidney compensation occurs with an exchange of hydrogen ions for potassium leading to a worsening of the metabolic alkalosis. The resulting metabolic condition is hypochloremic, hypokalemic, metabolic alkalosis. Ultrasound is the diagnostic modality of choic

A 32-year-old woman presents with abdominal pain, nausea, vomiting, and change in skin color for 6 days. She states that she had unprotected intercourse 4 weeks ago. Which one of the following tests indicates acute infection with hepatitis B as the cause of the patient's symptoms? A) Antibody to hepatitis B e antigen (Anti-HBe) B) Antibody to hepatitis B surface antigen (Anti-HBs) C) IgG antibody to B core antigen (Anti-HBc-IgG) D) IgM antibody to B core antigen (Anti-HBc-IgM)

Correct answer: D) IgM antibody to B core antigen (Anti-HBc-IgM) Explanation: This patient presents with symptoms consistent with acute Hepatitis B virus (HBV) infection. HBV is primarily transmitted through parenteral exposure (needle stick, intravenous drug use) or through unprotected intercourse. Transmission through blood transfusion is rare due to advances in screening techniques. Acute viral hepatitis presents with malaise, fever, anorexia, nausea, vomiting, abdominal discomfort, and diarrhea. Often, jaundice leads patients to consult a physician. Fulminant hepatitis is characterized by the acute onset of hepatic failure and encephalopathy over a short period of time (usually days). Measurement of hepatic enzymes can demonstrate 10- to 100-fold elevations of aspartate aminotransferase (AST) and alanine aminotransferase (ALT) levels. ALT is usually elevated to a greater degree than AST in viral hepatitis (the reverse is usually true in alcoholic hepatitis). Hyperbilirubinemia ca

An 82-year-old man has a myocardial infarction. On day 2 of his ICU admission, he reports severe left-sided abdominal pain. The nursing staff report his last bowel movement was bloody. His medical history is significant for Parkinson disease, hypertension, hyperthyroidism, hypercholesterolemia, and myocardial infarction. His abdomen is tender, and the left lower quadrant bowel sounds are diminished. Which of the following is the most likely diagnosis? A) Cholecystitis B) Diverticulitis C) Intes

Correct answer: D) Ischemic colitis Explanation: This history is most consistent with acute ischemic colitis. Ischemic colitis, the most common cause of bowel ischemia, represents a circulatory problem that occurs mainly in the older population. Some causes are septic or hemorrhagic shock, hypotension (myocardial infarction), embolic disease (atrial fibrillation, valvular disease), vasoconstriction, or a complication of aortic aneurysm repair. Symptoms include acute, severe abdominal pain and tenderness (usually left-sided) with bloody stools. Fever and peritoneal signs may indicate infarction. The acute phase progresses to a bowel paralytic phase in which bowel sounds are hypoactive or absent, the abdomen distends, and bloody stool passage ceases. Both of these phases may culminate in a bowel shock phase, in which fluid losses ensue, resulting in shock, hypotension, dehydration, and metabolic acidosis. Clinical suspicion may require computed tomography or endoscopic evaluation to sec

A 4-year-old boy presents to your office with watery nonbloody diarrhea for the last three months. He has occasional abdominal pain prior to the onset of the diarrhea and his mother noted worsening flatulence, especially after meal times. Mom denies any recent travel or camping and the patient does not go to school or daycare. On exam, he is 60th percentile for weight and height (similar to his last well-child check), appears well, and has a normal abdominal exam. What is the most likely diagnos

Correct answer: D) Lactase deficiency Explanation: Congenital lactase deficiency is rare and is associated with symptoms occurring on exposure to milk. However, fewer than 50 cases have been reported worldwide. Secondary lactose intolerance follows small bowel mucosal damage from conditions such as celiac disease and rotavirus infection and is usually transient, improving with mucosal healing. Patients can develop diarrhea, abdominal cramps, abdominal pain, and flatus following ingestion of milk products. Symptom severity depends on the amount of lactose ingested and the fat content of the product (skim milk empties from the stomach faster causing more symptoms). It is not associated with a failure to thrive. Lactase deficiency can be diagnosed by H2-breath test or by measurement of lactase activity in mucosa tissue retrieved by small bowel biopsy. Diagnostic testing is not mandatory, and often simple dietary changes that reduce or eliminate lactose from the diet relieve symptoms. Tr

Which of the following findings seen on rectal examination is most consistent with a concomitant systemic process? A) Anal fissure with bleeding B) Anal fissure with deep ulcer C) Anterior midline anal fissure D) Lateral anal fissure

Correct answer: D) Lateral anal fissure Explanation: A lateral anal fissure is concerning for an underlying systemic illness in a patient. Most anal fissures occur in the midline. Fissures that occur in other locations should raise concern for diseases like Crohn disease, HIV, leukemia, tuberculosis, or syphilis. Most commonly, fissures develop from a superficial tear in the anoderm when hard stool is passed, often associated with constipation. They occur in areas of weakness of the muscle fibers, most commonly in the posterior midline but also in the anterior midline. Patients report minor rectal bleeding and significant pain with defecation.

Which of the following statements is most accurate regarding health maintenance for a patient with chronic pancreatitis? A) Abstinence from alcohol will improve symptoms but will not decrease mortality B) Enteric coated pancreatic enzymes require co-administration with an H2 antagonist C) Long chain triglycerides can provide extra calories in patients with weight loss D) Malabsorption of fat soluble vitamins may occur but is rarely clinically symptomatic

Correct answer: D) Malabsorption of fat soluble vitamins may occur but is rarely clinically symptomatic. Explanation: Inflammation of the pancreas which is persistent and causes structural damage and eventual decline in pancreatic function is known as chronic pancreatitis. The cause of chronic pancreatitis may be obstructive or necrotic in nature. In obstructive pancreatitis, ductal stone formation leads to chronic inflammation which causes progressive pancreatic fibrosis and eventual atrophy. In necrotic pancreatitis, recurrent attacks of acute pancreatitis lead to necrotic tissue, which is eventually replaced by fibrotic tissue. Smoking and alcohol consumption are two main risk factors for pancreatitis. Other etiologies of pancreatitis include autoimmune disease, genetic disorders, and tumors. Symptoms of chronic pancreatitis are abdominal pain, nausea, vomiting and weight loss. Steatorrhea develops later in the disease course after the pancreas has lost much of its exocrine function. Diagnosis can be made via visualizations of calcifications on plain film or CT scan, magnetic resonance cholangiopancreatography, endoscopic ultrasound, or pancreatic function tests (such as stimulation of the pancreas and subsequent collection of duodenal fluid for analysis). Treatment of chronic pancreatitis involves lifestyles changes, pain management, and administration of pancreatic enzymes. Cessation of alcohol intake and smoking is imperative. Small, low-fat meals should be advised. Pain can be managed with tricyclic antidepressants, narcotics, or occasional hospitalizations to keep the patient nothing by mouth (NPO) for a short duration and allow pancreatic rest. Pancreatic enzymes are prescribed for patients with chronic pancreatitis to break the feedback loop of cholecystokinin release and pancreatic exocrine stimulation. Medium chain triglycerides are easily absorbed and can provide extra calories for patients suffering weight loss. Malabsorption of vitamins due to pancreatic exocrine malfunction may occur but is usually not clinically symptomatic. Replacement of the fat soluble vitamins and cyanocobalamin may be necessary for patients with substantial steatorrhea. Patients who fail medical therapy may be candida

A 50-year-old man with a history of hyperlipidemia and a recent diagnosis of colon cancer presents to the office. He is scheduled for surgical resection in 6 weeks. He is in no apparent distress today in the office. He currently takes atorvastatin 10 mg daily. Vital signs show a heart rate of 80 bpm, blood pressure 127/75 mm Hg, respiratory rate 18/minute, and oxygen saturation of 98% on room air. Physical exam reveals a normal cardiac exam with audible S1 and S2 without murmur. Lungs are clear

Correct answer: D) Monitoring for residual disease Explanation: Colorectal cancers account for about 10% of cancer-related deaths in the United States per year. They usually arise from mutations in adenomatous polyps. Risk factors can be genetic or environmental, with genetic susceptibilities carrying the most increased risk. Genetic risk factors include first-degree relatives with colon cancer, a history of breast or gynecologic cancer, and familial polyp syndromes. Environmental risk factors include hypercholesterolemia, coronary artery disease, low-fiber and high-animal-fat diets, obesity, tobacco or alcohol use, long-term red meat ingestion, and low socioeconomic status. Patients may be asymptomatic and diagnosed via screening or present with intestinal obstruction, perforation, or acute bleed. There may also be a change in bowel habits, hematochezia, abdominal pain, or blood per rectum. Diagnosis is made via biopsy through colonoscopy or surgery. While routine laboratory tests d

An 83-year-old man from a nursing home is sent for evaluation of abdominal distention and vomiting. Nursing home records report no bowel movement for 2 days and no fevers. His X-ray is shown above. On CT scan, no obstructing lesion is identified. Which of the following may be beneficial in relieving this condition? A) Barium enema B) Exploratory laparotomy C) Magnesium citrate D) Neostigmine

Correct answer: D) Neostigmine Explanation: The X-ray demonstrates massive dilation of the colon through the cecum consistent with a large bowel obstruction. However, CT scan does not show any obstructing lesion. Therefore, this is a pseudo-obstruction, also known as Ogilvie syndrome. The exact mechanism of the development of pseudo-obstruction is unknown but suspected to result from malfunction of the autonomic control of the bowel. Patients often have multiple other comorbid conditions and risk factors include nursing home residence, anticholinergic medication, severe electrolyte disturbance, narcotic exposure, or a history of spine or retroperitoneal trauma. This is a diagnosis of exclusion. Patients may first be treated with a rectal tube and sigmoidoscopy and cared for conservatively in the hospital. Neostigmine may be used as a pharmacologic intervention as it is an acetylcholinesterase inhibitor. It is not recommended in patients with a heart rate < 60 bpm, systolic pressure < 90 mm Hg, or active bronchospasm. Patients receiving neostigmine should be on a cardiac monitor and atropine should be at the bedside. A nasogastric tube is also an option and is used in mild to moderate cases of Ogilvie syndrome. A nasogastric tube should be avoided with signs of ischemia or perforation or when the cecum is > 12 cm in diameter.

A 66-year-old man presents with nausea, intermittent abdominal cramping, and frequent diarrhea for the past three days. He was recently hospitalized for pneumonia and was discharged one week ago. He was feeling better until yesterday. Physical examination reveals mild lower abdominal tenderness but is otherwise unremarkable. Of the following choices, which is the most appropriate initial treatment? A) Intravenous vancomycin B) Oral ciprofloxacin C) Oral diphenoxylate and atropine D) Oral vancomycin

Correct answer: D) Oral Vancomycin Explanation: Either oral fidaxomicin or oral vancomycin is recommended for all initial episodes of C. difficile colitis. Diarrhea in excess of 10 stools per day, leukocytosis greater than 15,000/μL, fever, and signs of dehydration indicate severe disease and usually warrant hospitalization. C. difficile is often precipitated by antibiotic use, especially aminopenicillins, fluoroquinolones, cephalosporins, and clindamycin. After colonization, toxins are produced by the gram-positive, spore-forming bacillus. Relapses are common after treatment and occur in approximately 25 percent of all treated patients.

A 60-year-old woman with a history of heart failure presents with substernal chest pain that awoke her from sleep. She has dysphagia when swallowing solids and odynophagia when swallowing both solids and liquids. She reports a history of chest pain in the past but states that this pain is different in that it is sharp and occurs only after swallowing. Her medications include metoprolol succinate, furosemide, potassium chloride, simvastatin, isosorbide dinitrate, and levothyroxine. Which of the f

Correct answer: D) Questioning her about her medication use Explanation: Pill esophagitis is caused by a pill becoming trapped in a collapsed esophagus. Questioning the patient about when and how she takes her medications may elicit information suggesting pill esophagitis which is the most likely cause of her chest pain. The position of the patient, the size of the medication, and the amount of fluid ingested with the medication are important determinants of the risk of medication-induced esophagitis. Ingestion of a pill immediately prior to sleep is also associated with an increased risk of medication-induced esophagitis as both salivation and swallowing frequency are markedly reduced during sleep. Patients with medication-induced esophagitis usually present with retrosternal pain or heartburn, odynophagia, and dysphagia. In some cases, the pain may be so severe that swallowing saliva is difficult. Rarely, patients may have hematemesis, abdominal pain, and weight loss. Many medicati

A 29-year-old woman with no significant medical history presents to her primary care office to further discuss a chest X-ray that was done during a recent emergency department visit. The patient reports she was seen in the emergency department 2 weeks ago for cold-like symptoms, which have since resolved. At this time, the patient states she is back to her baseline and reports none of the following symptoms: cough, congestion, heartburn, or chest pain. The chest X-ray from the emergency departme

Correct answer: D) Reassurance and observation Explanation: This patient has a sliding hiatal hernia, which occurs when the lower esophageal sphincter moves above the diaphragm and leads to dysfunction of the gastroesophageal junction reflux barrier. Most sliding hiatal hernias are asymptomatic, such as in the patient above. However, if patients do develop symptoms, they are similar to gastroesophageal reflux disease (GERD), such as heartburn, belching, and regurgitation. For this patient, the diagnosis was made by a chest radiograph, which showed a retrocardiac air-fluid level. However, diagnosis can be confirmed via endoscopy, esophageal manometry, or barium swallow, which is the most sensitive test. For all asymptomatic sliding hiatal hernias, the treatment of choice is reassurance and observation. For patients with sliding hiatal hernias who have symptoms, the treatment of choice is to treat the GERD-like symptoms with diet and lifestyle modifications, antacids, histamine H2 rece

A 43-year-old woman presents with right upper quadrant abdominal pain for 3 weeks. She states that she intermittently gets sharp pain that occurs after eating and is associated with nausea and occasionally vomiting. The pain lasts for 10-15 minutes, and then spontaneously improves. Currently, she has no pain. Her vital signs and bloodwork are normal. A right upper quadrant ultrasound is shown above. What management is indicated? A) Administer antibiotics and admit for observation B) Admit patient for cholecystectomy C) Obtain CT scan of the abdomen and pelvis D) Referral for surgical consultation and pain medication as needed

Correct answer: D) Referral for surgical consultation and pain medication as needed. Explanation: This patient presents with biliary colic as a result of cholelithiasis (gallstones). Cholelithiasis is a common disorder affecting 20% of women and 8% of men. It usually presents with intermittent right upper quadrant (RUQ) or epigastric pain, which may radiate to the base of the right scapula or shoulder. The pain is often associated with nausea and vomiting. Attacks are usually self-limited and often occur after eating. The theory is that the gallbladder contracts in response to food and contraction against the stone causes pain. Laboratory studies are usually non-contributory but can be useful in ruling out common bile duct obstruction (elevated bilirubin and alkaline phosphatase) and pancreatitis (elevated lipase) that can occur when gallstones move out of the gallbladder. Plain radiographs are not helpful in the diagnosis, as only 10% of stones have enough calcium to be visualized. Ultrasound, on the other hand, is rapid and highly sensitive in the diagnosis of cholelithiasis and cholecystitis. In patients who present with typical symptoms of biliary colic with an ultrasound supporting this diagnosis, management should focus on supportive care (depleting fluids and electrolytes lost in vomiting), pain control, and ruling out other causes of abdominal pain. Once the symptoms are managed, patients can be discharged with appropriate follow-up. Advising the avoidance of fatty foods and alcohol can reduce the recurrence of symptoms. The definitive management of symptomatic cholelithiasis is cholecystectomy.

Twenty-four hours after eating a salad containing bean sprouts, a 25-year-old man became ill with fever, abdominal pain and diarrhea. Which of the following organisms is the most likely cause of his symptoms? A) Clostridium perfringens B) Cryptosporidium parvum C) Enterotoxigenic Escherichia coli D) Salmonella enterica

Correct answer: D) Salmonella enterica Explanation: Foodborne illness should always be on the differential when a patient presents with gastrointestinal symptoms such as nausea, vomiting, diarrhea, abdominal pain and fever. A thorough history provides important information in diagnosing foodborne illness and should include description of clinical symptoms, exposure to high-risk types of food and the duration of time between consumption of the infected food and onset of symptoms. Salmonella enterica is a gram-negative bacterium that causes inflammatory diarrhea. Signs and symptoms of inflammatory diarrhea may include the presence of blood or mucus in the stool, severe abdominal pain and fever. Salmonellosis is typically associated with consumption of raw meat or poultry, but can also be associated with fresh produce such as bean sprouts, tomatoes, lettuce, and melons. The illness is self-limited and treatment generally is with fluid replacement, although antibiotics are used in certai

A 46-year-old obese man presents to the clinic complaining of recurrent heartburn. He reports compliance with optimal medical management and dietary modifications and has been unsuccessful at losing weight. Which of the following is a contraindication to surgical intervention and should be ruled out before proceeding? A) Benign esophageal strictures B) Gastroparesis C) Hiatal hernia D) Scleroderma

Correct answer: D) Scleroderma Explanation: Surgical fundoplication may ultimately be necessary, however, alternative diagnoses should be investigated prior to proceeding. Scleroderma presents a contraindication to surgery for gastroesophageal reflux disease due to the high incidence of postoperative dysphagia. One of the most serious complications of gastroesophageal reflux disease is Barrett esophagus, which manifests with alarm symptoms and undergoes malignant transformation into adenocarcinoma.

A 28-year-old man presents to the emergency department with severe upper abdominal pain, nausea, and vomiting for 8 hours. Vital signs today include a heart rate of 112 bpm, blood pressure of 125/85 mm Hg, respiratory rate of 20/minute, oxygen saturation of 98% on room air, and temperature of 99.2°F. Physical examination reveals a man in acute distress with epigastric tenderness but no abdominal rigidity, rebound tenderness, or bruising. Laboratory findings include a white blood cell count of 7

Correct answer: D) Serum lipid profile Explanation: Acute pancreatitis is a disease process marked by pancreatic inflammation caused by pancreatic enzymes that have leaked from damaged pancreatic cells autodigesting the pancreas. The most common causes of acute pancreatitis are gallstones and alcohol use. However, other causes include hypertriglyceridemia, endoscopic retrograde cholangiopancreatography, medication adverse effects, autoimmune disease, and idiopathic disease. A serum lipid profile should be obtained in patients with acute pancreatitis to assess for hypertriglyceridemia. The classic presentation is an abrupt onset of severe upper abdominal pain with nausea and vomiting. The pain often radiates to the back. Exam findings include epigastric abdominal tenderness. Hemorrhagic pancreatitis can cause Cullen sign (periumbilical bruising) or Grey Turner sign (bruising in bilateral flanks) in a minority of cases. Acute pancreatitis requires two of the three criteria for diagnosis

Which of the following supports the diagnosis of biliary colic? A) An ultrasound that shows an empty gallbladder and duct B) Pain that is relieved by eating food C) Radiation of abdominal pain to the lumbar region D) Steady abdominal pain localized in the right upper quadrant

Correct answer: D) Steady abdominal pain localized in the RUQ. Explanation: The term colic is misleading in the diagnosis of biliary colic. Most patients with biliary colic have steady pain, not colicky, in the right upper quadrant or epigastrium. The pain of biliary colic is thought to be caused by contraction of the gallbladder, with transient obstruction of the cystic duct or common bile duct by the stone. With continued obstruction, inflammation of the gallbladder wall leads to cholecystitis. Physical exam in biliary colic usually reveals mild tenderness to palpation in the right upper quadrant. Lab tests are usually normal. The diagnosis is made clinically and by demonstrating stones in the gallbladder on ultrasound. Plain films have a limited role in detecting gallstones because less than 10% of gallstones contain calcium. Management is usually supportive.

A 47-year-old man with chronic low back pain presents to the emergency department with epigastric pain for 3 weeks. The pain is burning, without radiation, and occurs 2-3 hours after eating. Vital signs are normal. He takes ibuprofen and naproxen for his back pain. What management is indicated? A) Admit for endoscopy B) Start esomeprazole and refer for outpatient evaluation C) Start ranitidine and refer for outpatient evaluation D) Stop ibuprofen and naproxen and refer for outpatient evaluat

Correct answer: D) Stop ibuprofen and naproxen and refer to outpatient evaluation Explanation: The patient presents with clinical gastritis, likely secondary to the use of nonsteroidal anti-inflammatory drugs (NSAIDs). Technically, gastritis is a histologic diagnosis indicating inflammation of the gastric mucosa and can only be definitively made after endoscopy and biopsy. Clinicians, however, refer to the symptoms of dyspepsia (pain in the upper abdomen that presents as bloating or heartburn) as gastritis. The most common cause of gastritis is Helicobacter pylori infection, but it also commonly results from medications, including salicylates and NSAIDs (ibuprofen, naproxen). Acute gastritis typically presents with abdominal pain located in the epigastric area that is burning in nature and can also have bloating or nausea associated with it. Treatment should begin with removal of any possible inciting agents including alcohol, smoking, steroids, or NSAIDs.

A 59-year-old man with a history of hypertension, dyslipidemia, and chronic low back pain presents for evaluation of sudden-onset abdominal pain that radiates to his right shoulder. His current medication regimen includes enalapril, propranolol, aspirin, ibuprofen, and atorvastatin. He describes intense, diffuse abdominal pain that began 2 hours ago and has since decreased in severity. Vital signs are HR 133 bpm, RR 20/min, BP 102/82 mm Hg, T 95.7°F, and SpO2 95% on room air. Physical exam reve

Correct answer: D) Surgical consultation Explanation: Peptic ulcer disease is a gastrointestinal disorder most commonly caused by nonsteroidal anti-inflammatory drug use and Helicobacter pylori infection that results in ulcerative lesions within the stomach and duodenum. Free perforation may occur when a lesion ulcerates through the gastrointestinal wall. Other complications include gastric outlet obstruction, penetration, and bleeding. Patients with comorbid disease, advanced age, and more severe physiologic insult (e.g., hypotension, metabolic acidosis, acute kidney injury, hypoalbuminemia) are more likely to have poor outcomes from complicated peptic ulcer disease. The clinical manifestations of ulcer perforation can be described in three phases. The initial phase occurs within 2 hours of onset and is characterized by sudden, severe, diffuse abdominal pain that may radiate to the right shoulder or bilateral shoulders. Physical examination in this phase reveals tachycardia, a weak

A 12-month-old boy is brought to the emergency department by his mother for three days of intermittent episodes of inconsolable crying. This morning the boy became lethargic and difficult to awaken. He has had several episodes of nonbilious vomiting and grossly bloody stools. His vital signs are T 38.3°C, HR 140, BP 80/50, RR 22. On examination, he is lethargic but diffusely tender on abdominal palpation with involuntary guarding. His abdominal X-ray reveals free air under the diaphragm. Which

Correct answer: D) Surgical reduction Explanation: Intussusception is the invagination of a bowel segment into a distal segment of bowel. It is the most common abdominal emergency in early childhood and is most common in males and in children under two years of age. The classic presentation of intussusception is intermittent abdominal pain, drawing up of the legs towards the abdomen, bloody "currant-jelly" stools mixed with mucus, and a right-sided sausage-shaped abdominal mass. Patients may also present only with progressive lethargy and altered consciousness, a presentation which is often mistaken for sepsis. The diagnostic modality of choice in suspected intussusception with uncertain presentation is ultrasound. In patients with high suspicion for intussusception air-contrast enema is both diagnostic and therapeutic. Contraindications to the use of air-contrast enema include hemodynamic instability with shock, free air under the diaphragm, and peritonitis. Children with these

A 72-year-old man presents with progressive dysphagia. He initially had difficulty swallowing solid foods but is now having difficulty swallowing liquids. An esophagogastroduodenoscopy reveals a fungating mass on the middle one-third of the esophagus. Biopsy of the lesion is positive for squamous cell carcinoma. Which of the following historical features does this man most likely have? A) Exposure to industrial dyes B) Gastroesophageal reflux disease C) Lynch syndrome D) Tobacco use

Correct answer: D) Tobacco use Explanation: Tobacco use is one of the most common risk factors for the development of squamous cell carcinoma (SCC) of the esophagus. The most common types of esophageal cancer are SCC and adenocarcinoma. Barrett metaplasia, from chronic, untreated gastroesophageal reflux disease (GERD), is the most common cause of esophageal adenocarcinoma. Tobacco smoking and alcohol consumption are the most common causes of esophageal SCC. Adenocarcinoma is most commonly seen in White men, while SCC is most commonly seen in Black and Asian individuals. Adenocarcinoma and SCC have a similar clinical presentation of progressive dysphagia. Difficulty swallowing solid food, such as meat, bread, or fruit, typically precedes dysphagia to liquids. Weight loss is commonly noted. Esophagogastroduodenoscopy with biopsy is the most commonly used diagnostic tool for establishing the diagnosis and determining staging. Most esophageal SCC is located in the middle thoracic esophagus. Adenocarcinomas are most commonly found in the distal esophagus and the gastroesophageal junction. CT and PET may be used for determining lymph node involvement. Lymph node involvement or metastases is associated with poor prognosis. Treatment varies by disease stage and may include surgical resection, chemotherapy, and radiation therapy. -GERD (B) is a risk factor for the development of esophageal adenocarcinoma. Patients with long-standing GERD have the greatest risk for adenocarcinoma development. Endoscopic screening is recommended for patients with Barrett esophagus. Smoking also increases the risk of adenocarcinoma.

A 66-year-old man presents with nausea and recurrent vomiting. His spouse accompanies him and thinks the patient's voice has changed. His medical history is significant for heavy alcohol and tobacco use, hypertension, COPD, and celiac disease. During examination, his voice is raspy and hoarse. You order a chest and neck CT scan, which reveals a mass. Which of the following is the most likely location of this mass? A) Gastric cardia B) Gastric pylorus C) Lower one-third of the esophagus D) U

Correct answer: D) Upper two-thirds of the esophagus Explanation: Esophageal cancer most often affects patients 50-70 years of age, with a male to female ratio of 3:1. Squamous cell carcinoma is strongly associated with chronic alcohol use and smoking and is also more common in the Black population. In the majority of cases, adenocarcinoma develops from Barrett esophagus. A diet low in fruits and vegetables is a risk factor, as is obesity. Patients present with solid food dysphagia that progresses over weeks to months. Weight loss is common. Other symptoms include cough on swallowing and chest or back pain. Voice hoarseness is seen with invasive disease. Laryngeal function depends on the integrity of the recurrent laryngeal nerves. With esophageal squamous cell carcinomas, a growing mass can encroach upon or envelope these nerves, leading to altered function with resultant voice changes. Definitive diagnosis is made by biopsy via endoscopy. CT scan can also identify the mass and is

A 56-year-old man with a 10-year history of alcohol use disorder presents to the emergency room with nausea and dull, epigastric pain that radiates to the back for the past 2 hours. Which of the following lab values is associated with a poor prognosis for the suspected diagnosis? A) Aspartate aminotransferase 200 units/L B) Glucose 172 mg/dL C) Serum lactate dehydrogenase 300 units/L D) White blood cell count 18,000

Correct answer: D) WBC count 18,000 Explanation: Acute pancreatitis is most commonly caused by obstruction of the biliary ducts or excessive alcohol use. Other causes of acute pancreatitis include hypercalcemia, hyperlipidemia and abdominal trauma. The signs and symptoms of acute pancreatitis include steady, dull or sharp epigastric pain with radiation to the back. Pain is often relieved by leaning in a forward position. Ranson's criteria is used to assess the severity of illness and % mortality. If 3 or more of the criteria are met, there is an increased risk for pancreatic necrosis. The following criteria, if met within the first 24 hours of presenting symptoms, is used to evaluate the severity of pancreatitis: age > 55, white blood cell count > 16,000, blood glucose > 200 mg/dL, serum lactate dehydrogenase > 350 units/L, and aspartate aminotransferase > 250 units/L. Note that amylase and lipase levels are not considered part of this criteria. The diagnosis of acute pancreatitis r

You are seeing a 4-week-old boy with vomiting. Which of the following findings is most concerning for pyloric stenosis as an underlying cause? A) Bilious emesis B) Hyperkalemic, Hypochloremic metabolic acidosis C) Vomitus with the appearance of "curdled milk" D) Weight loss

Correct answer: D) Weight loss Explanation: Infantile pyloric stenosis should be considered in a neonate with vomiting and weight loss. The classic presentation of infantile pyloric stenosis is in a neonate one to three months of life with progressively forceful, non-bilious emesis. Males predominate over females in 5:1 ratio. A history of erythromycin exposure should also raise suspicion. Individuals with infantile pyloric stenosis typically display immediate hunger after vomiting. The classic exam reveals an olive-shaped mass in the epigastrium with peristaltic waves visible in the left upper quadrant. Classic presentation also includes dehydration and a hypochloremic, hypokalemic metabolic alkalosis. It should be noted that despite the classic "textbook" presentation of infantile pyloric stenosis, patients tend to present earlier, are less dehydrated, and have fewer electrolyte abnormalities. This is thought to be due to improved awareness and imaging capabilities.

A 61-year-old man presents to the clinic for evaluation. His medical history includes chronic obstructive pulmonary disease with an ongoing 25 pack-year history of tobacco use. The patient takes albuterol as needed in addition to his daily medications, which include salmeterol and budesonide. Following a thorough history and physical exam, the patient undergoes diagnostic colonoscopy, which identifies a polypoid, endoluminal mass in the ascending colon. Subsequent biopsy indicates colorectal ade

Correct answer: D) progressive fatigue and weight loss Explanation: Colorectal cancer results from the malignant transformation of polyps within the large bowel. It is the second most commonly diagnosed cancer in women and the third most common in men worldwide. Most early-stage carcinomas are asymptomatic and diagnosed as a result of screening. Symptomatic colorectal cancer is most commonly caused by growth of the tumor into the lumen or adjacent structures and presents with blood per rectum, abdominal pain, and anemia. Changes in bowel habits may also be reported. Iron deficiency anemia with associated progressive fatigue and weight loss is commonly found in patients with right-sided colorectal cancer. Metastatic disease can also result in signs and symptoms such as right upper quadrant pain, abdominal distention, early satiety, supraclavicular lymphadenopathy (Virchow node), or a periumbilical nodule (Sister Mary Joseph nodule). The preferred screening method is colonoscopy in t

An 18-month-old boy is brought to the emergency department by his mother for two days of intermittent episodes of inconsolable crying. During these episode, the child is noted to stop playing, lay down, and draw his legs up to his chest. His mother denies any emesis or bloody bowel movements. Physical examination and vital signs in the emergency department are normal. Which of the following is the most appropriate imaging modality for the confirmation of a diagnosis of intussusception in this pa

Correct answer: D) ultrasonography Explanation: Intussusception is the invagination of a bowel segment into a distal segment of bowel. It is the most common abdominal emergency and cause of intestinal obstruction in early childhood. Intussusception is most common in males and in children under two years of age. The majority of cases of pediatric intussusception are idiopathic, but risk factors include conditions that cause a lead point for intestinal telescoping, including Meckel's diverticulum, Henoch-Schonlein purpura, intestinal tumors, rotavirus vaccine, and recent viral infection. The classic presentation of intussusception is intermittent abdominal pain, drawing up of the legs towards the abdomen, vomiting, bloody "currant-jelly" stools mixed with mucus, and a right-sided sausage-shaped abdominal mass. In reality, however, the triad of intussusception - pain, bloody stools and abdominal mass - is rarely present. Children may present only with progressive lethargy and alter

A 65-year-old man presents to the clinic with several weeks of dull, gnawing epigastric pain that is usually relieved by eating. He has been taking omeprazole for five weeks, but has had no improvement. His only other daily medication is ibuprofen for knee pain. An abdominal exam is normal and fecal occult blood testing is negative. The next best step in management includes which of the following diagnostic tests? A) Abdominal CT B) Barium upper gastrointestinal series C) H. pylori fecal antig

Correct answer: D) upper endoscopy Explanation: Upper Endoscopy is the next best step in management of this patient's epigastric pain because it will aid in visualization of a gastric or peptic ulcer and allow for biopsy if necessary. Based on the location and description of this patient's pain, it is likely that he has peptic ulcer disease (PUD), specifically duodenal as the pain is relieved by eating. Patients with PUD typically complain of epigastric pain that is aching or "hunger-like" and may be relieved with food or antacids for 2-4 hours. It is not uncommon for patients to be awakened with pain, particularly with duodenal ulcers. This pain may persist on a regular basis for weeks, months, or years; a change in character of pain may indicate a perforation. Patients with gastric ulcers may also complain of nausea and anorexia. However, any significant vomiting or weight loss may indicate a more serious gastric obstruction or malignancy. In uncomplicated PUD, laboratory fi

A 52-year-old man presents to the clinic with concerns over his worsening dysphagia. He describes difficulty with both solids and liquids and occasionally regurgitates bland, undigested food. He has been diagnosed previously with gastroesophageal reflux disorder, which was unresponsive to a 4-week trial of omeprazole. His vitals include a HR of 83 bpm, RR of 16/min, BP of 128/88 mm Hg, T of 98.7°F, and SpO2 100% on room air. Physical exam reveals a well-appearing man in no acute distress withou

Correct answer: E) Pneumatic balloon dilation Explanation: Achalasia is an esophageal motility disorder thought to result from progressive degeneration of ganglion cells in the myenteric plexus of the esophageal wall. This defect causes the lower esophageal sphincter (LES) to remain tonically contracted and results in a loss of peristalsis within the distal esophagus. Achalasia can be primary, with still unknown etiology, or secondary to diseases such as amyloidosis, sarcoidosis, neurofibromatosis, or eosinophilic esophagitis, among others. Although this condition may present at any age, patients are most frequently diagnosed between ages 25 and 60. Clinical presentation predominantly includes dysphagia for solids and liquids and regurgitation of bland, undigested food or saliva. Additionally, patients may report chest pain, heartburn, and difficulty belching. Upper endoscopy may reveal retained food in the esophagus or an unusually increased resistance to passage of the scope throug

A 30-year-old woman with a history of Crohn disease presents to the emergency department with a report of rectal pain for about 2 weeks. She states she has pain with sitting and bowel movements and has noticed some discharge from her rectal area. She reports no fevers. Her vital signs show a heart rate of 80 bpm, blood pressure 119/75 mm Hg, respiratory rate 17 breaths/min, and SpO2 of 98% on room air. On physical exam, her abdomen is soft with mild tenderness to palpation in the suprapubic region. On rectal exam, there is scant purulent drainage with a small pustule noted in the perianal region. There is a palpable cord felt from the external surface to the anal canal. Imaging and labs are pending. What is the most appropriate management for the suspected diagnosis? A) Empiric antibiotics B) Incision and drainage C) Sitz baths D) Stool test for ova and parasites E) Surgical referral for evaluation

Correct answer: E) Surgical referral for evaluation Explanation: Perianal disease can affect up to one-third of patients with Crohn disease and may include hemorrhoidal tags, anal stricture, perirectal abscess, and anorectal fistula. Anorectal fistula develops after an anal abscess ruptures or is drained. It forms via a tract connecting the abscess to the perirectal skin. Patients will often present with an anal abscess that is not healing and reports of intermittent rectal pain, especially with defecation, as well as with sitting or other physical activity. On physical exam, there may be purulent drainage or a pustule-type lesion in the perianal area, and excoriation and inflammation of the external skin may be present. A cord from the external opening to the anal canal may also be palpable. The fistula can often be probed to determine its depth, but care should be taken to avoid perforation. Evaluation includes MRI, endosonography, or both to determine the exact anatomy of the fistula, followed by surgical referral for evaluation and further treatment as necessary next steps. As initial treatment, a silk tie may be placed through the fistula tract to allow for inflammation to subside while still allowing drainage. After the inflammation has subsided, a fistulotomy can be performed in simple fistulas or an anorectal flap with drainage catheter in more complex tracts or those located above the anal sphincter.

A 65-year-old man presents to the primary care clinic with trouble swallowing. His symptoms initially began about 2 months ago when he started having trouble swallowing solids and he had to chew longer and eat slower. In the last 2 weeks, he has noticed he is having trouble swallowing liquids, too. He also has odynophagia and has lost about 10 lbs in the last 2 months. He smokes and has a 40-year pack history, along with a history of hypertension and high cholesterol. He takes lisinopril 10 mg a

Correct answer: E) Upper GI endoscopy Explanation: The above scenario is suspicious for esophageal cancer. There are two main types of esophageal cancer (squamous cell carcinoma and adenocarcinoma), and both are typically seen in patients over the age of 50 who have a history of chronic tobacco or alcohol use. Intestinal metaplasia (secondary to GERD) is a risk factor for adenocarcinoma of the esophagus. Male sex and obesity are also risk factors for esophageal cancer. Patients with esophageal cancer often report progressive dysphagia to solid food, weight loss, and painful swallowing, though symptoms can be subtle at first. Physical exam is usually normal in patients with esophageal cancer. An upper gastrointestinal endoscopy with biopsy is the best test to evaluate for esophageal malignancy in this patient. If this patient reported a history of radiation, previous surgery due to esophageal or laryngeal malignancy, or a complex stricture, then a barium swallow should be considered.

Which of the following tests can be used to confirm lactose intolerance? A) Anti-gliadin antibodies B) Hydrogen breath test C) Sweat test D) Urea breath test

Correct answer: Hydrogen breath test Explanation: A lactose hydrogen breath test will show a rise in breath hydrogen > 20 ppm within 90 minutes of ingesting 50 g of lactose when it is positive for lactase deficiency. This test will be positive in 90% of patients with lactose malabsorption. Common causes of false-negative results are recent use of oral antibiotics or recent high colonic enema. The diagnosis can usually be made on the basis of the history and improvement with dietary manipulation. Lactose intolerance is a prototypical carbohydrate malabsorption disorder. Individuals with the condition may experience diarrhea, cramps, abdominal pain, and flatus following the ingestion of milk products. Symptom severity depends on the amount of lactose ingested and the fat content of the product (i.e., skim milk empties from the stomach more quickly, causing more symptoms). Diagnostic workup may include confirming the diagnosis with the lactose hydrogen breath test and excluding other con


Set pelajaran terkait

Loud Pack 164 Kaplannzzzz UCSD Psych 164

View Set

Stephen F. Austin & the Empresarios of Texas

View Set

Pro Res - Stevenson 2019, 500 Questions

View Set

Intro to Nursing Concepts, PrepUs Ch 4

View Set

Islamic Studies Semester 2 Final Study Guide

View Set

Opera Composers during Baroque Period

View Set